Family Medicine Residency Personal Statement Examples

Family Medicine residency personal statement examples

It is a good idea to review family medicine residency personal statement examples as you  prepare for residency applications. Whether you are applying through  ERAS  (in the United States) or  CaRMS  (in Canada), your residency personal statement is one of the most important application components that you will need to submit. Residency program directors and admission committees want to get a sense of who you are as a person, the kind of  resident doctor  you can be, and why you have chosen your medical specialty. This is the information that they'll be hoping to get from your personal statement.

It should also be noted that even though family medicine is one of the least competitive medical specialties, all the best family medicine residency programs are relatively competitive. This means that you need to provide that information in a memorable way so that your personal statement stands out from the large pool of applicants.

In this blog, we share three outstanding family medicine residency personal statement examples  that will give you a better idea of what to include in your own statement. We also share some tips to help you write a strong personal statement for your  family medicine residency  application. 

>> Want us to help you get accepted? Schedule a free initial consultation here <<

Article Contents 12 min read

Family medicine residency personal statement example #1.

Dr. Jamie Clark was in the room on the day I was born. He was also there the first time I broke my leg jumping off the jungle gym, the first time I had to get stitches after I fell off my bike, on the day that I became an older sister, and on the day of my high school graduation when I twisted my ankle at the graduation party. Like many others in my hometown, my parents have built a long-lasting and trusting relationship with this person. To the extent that Dr. Jamie Clark feels like a member of my family, but he is really just our family physician. 

Most children hated going to the hospital, but I always enjoyed it. Partly because Dr. Clark always found a way to make it fun for me by joking the whole time and giving me lollipops even when I was a teenager, but also because I was always so fascinated by what he did even if I didn't understand it fully. I just knew that people always left his clinic feeling much better than when they came in. I also noticed that his waiting room was always full of different people of all age groups, and he seemed to know each of them personally. I would watch him talk to them at the hospital and even outside in social settings; people always seemed to seek him for advice, and they always appeared more at ease after talking to him. As someone who enjoys talking to people, hearing their stories, and connecting with them, that really appealed to me. 

It wasn't until later in high school that I started to understand what it truly meant to be a doctor, and I became interested in medicine. The more I learned about human anatomy, the more questions I had. One of my biology teachers advised me that if I really wanted to understand the systems of the human body, I should consider going to medical school. His statement stayed with me, and I started wondering if a career in medicine would be suitable for me. 

After high school, I took a gap year and spent six months of it shadowing Dr. Clark. During this time, I learned that family physicians are among the few specialists qualified to provide comprehensive health care for people of all ages, from newborns to seniors. As a family doctor, Dr. Clark got to work on a variety of different cases on a daily basis. On any given day, he could go from doing an echography for a pregnant woman to helping a child with a broken bone and then walking a cancer patient through the radiation therapy that they would be receiving over the next few months. 

On the day of my high school graduation, one of my teachers told me that three qualities produce great doctors: leadership, dedication, and compassion. 

I realized the importance of leadership and commitment as I led my high school varsity soccer team and participated in various other sports activities in college. I then used the leadership skills that I was developing to create a community outreach program. I coordinated with some members of my college student union to assist multiple homeless shelters in Township. We coordinated drives to the shelters once a month, and once every six months, one of the physicians and the local hospital would come along to provide check-ups, medical advice, and supplies. 

One particular man at one of the shelters always complained about having blisters on his feet. I had spoken to him on several occasions, and during one of our interactions, he mentioned that he had diabetes. Remembering what I was learning in school about the disease, I asked one of the doctors to speak with him during the next drive. I later found out that the man in question had to get an amputation of the foot due to diabetes complications. This experience taught me the importance of active listening and preventive care, which solidified my interest in a career in medicine. 

Upon graduation, I wanted to work to help people in remote communities, so I joined a public health service program and spent almost a year working as an assistant coordinator with a team of physicians who travel to underserviced communities to provide medical care. We saw an average of forty patients every day, and I was exposed to a broad range of pathologies. I learned so much about the wide range of diagnostic tools and treatment options physicians use daily. 

This experience helped me understand how valuable access to a primary care physician is, and it motivated me in a new way. The physicians with whom I had worked inspired me. Their patience and dedication to their patients filled me with admiration, and they taught me the particular importance of interpersonal skills and doctor-patient interaction. The patients that we had seen helped me improve my active listening skills and taught me that is important to build trust with your patients.

I started medical school with that motivation, and I remain focused on achieving my goal of becoming a family physician. I am especially interested in practicing in rural communities as I have seen first-hand how the lack of access to primary care has affected these underserved populations. As a family physician, you not only have the ability to help those who have been affected by illness, but also provide others with the information and treatment that they need to avoid preventable conditions or complications from manageable diseases. I believe that as a physician who is willing and able to help, I should do so while I can. 

Over the years, my experiences have helped me build the leadership skills and sense of compassion that I need to become a good doctor. I am passionate about the field of medicine and dedicated to providing not only adequate but excellent patient care. I am ready to invest my time and energy into my residency to further complete advanced training in family medicine and become the best family practitioner that I can be. 

Family Medicine residency personal statement example #3

I have been practicing medicine for just under ten years as a trained anesthesiologist, but it is time for me to become a family physician. I was always torn between the two specialties, but after gaining experience in both fields over the past few years, I can confidently say that a career in family practice would be better suited for me. 

Family practice is one of the only specialties that exposes physicians to a broad spectrum of pathology, allows me to form deep connections with patients of different ages, and gives them a chance to provide them with continuous care. While I know that family physicians do not get to do this with every patient, I love the idea of caring for patients from birth to death and building long-term friendships with them. All while still using my knowledge of pharmacology, physiology, and microbiology to help patients. 

Three years ago, when I started considering switching to a different specialty, I decided to volunteer at a family clinic in my hometown so that I could get some practical experience. I knew that the experience would either affirm my decision to leave anesthesiology and switch to family practice or convince me that I didn't need a change after all. Even though I was very limited in my duties and only interacted with patients under supervision, every day I spent at the clinic convinced me that family practice was ideal for me. 

I enjoyed speaking with the patients long enough to actually get to know them and make a genuine connection. I appreciated being able to counsel patients and provide preventive care. I especially enjoyed having to investigate and diagnose patients again. Starting with a chief complaint and a few symptoms, then looking back into the patient's history and examining them to come up with a differential diagnosis is like solving a puzzle. Something that I thoroughly enjoy doing in my spare time. 

Now that you know what a great family medicine residency personal statement should be like, let's go over a few things that you can do to improve the quality of your personal statement, and common mistakes to avoid if you want your residency personal statement to stand out.  

1. Do tell a story 

If your personal statement is just a bunch of facts about you put together, it will not be memorable. Remember that family medicine is one of the most popular medical specialties, so you are competing with a large number of qualified applicants. To stand out, you should structure and write your personal statement using a narrative. You want to evoke some sort of emotion from the reader so that they can connect with your story and, therefore, remember it. For example, take a look at the passage below from one of the essays that you read earlier. This student could have just as well said that they pay attention to details and actively listen to patients. Instead, they told a story that shows that they do and that is far more memorable.

One particular man at one of the shelters always complained about having blisters on his feet. I had spoken to him on several occasions, and during one of our interactions, he mentioned that he had diabetes. Remembering what I was learning in school about the disease, I asked one of the doctors to speak with him during the next drive. I later found out that the man in question had to get an amputation of the foot due to diabetes complications. This experience taught me the importance of active listening and preventive care, which solidified my interest in a career in medicine.  ","label":"Example:","title":"Example:"}]" code="tab1" template="BlogArticle">

It is imperative that you follow instructions when you are writing your personal statement. It is actually important to follow them as you prepare and submit all of your application components, such as the  ERAS letters of recommendation , for example. 

Unless your instructions clearly state otherwise, your personal statement should be 650 - 850 words. Students sometimes feel like this is not enough to provide specific information about their background, but it actually is. You want your residency personal statement to be informative but straight to the point. You do not want to bore the reader with additional details that can probably be found in your other application components. 

Remember that you were given instructions for a reason. Following them not only makes a good impression, but keeping your essay concise will actually help you stay on track and focus on providing only key information about your background, skills, and abilities. 

 3. Do start writing early

 Writing a strong personal statement for residency takes time. You need enough time to brainstorm and consult your medical student CV and other records to choose the experiences you learned the most from and want to discuss in your statement. Once you know how you want to structure your essay, you still need to write it, review it for edits, and polish it until it is as compelling as it can be. 

When you are crafting your Eras timeline or CaRMS timeline for Canadian applicants, we recommend that you plan at least six to eight weeks to write and edit your family medicine residency personal statement. 

4. Don't rehash your CV

The residency directors already have your  residency CV , and if you are applying to residency programs in the United States, then they also have your  ERAS experience section . In other words, they do not need an essay detailing your work or research experience. Instead, they want to get additional information about you. 

Focus on sharing what you learned from your different experiences. For example, if you are an International Medical Graduate (IMG), instead of telling the residency directors that you were able to  get electives and clinical experience in the US and Canada as an IMG , pick one of those clinical experiences and tell them what you learned from it and how it prepared you for your family medicine residency in your new country of residence.

Not sure what to include on your residency CV? This video can help:

5. Do proofread (multiple times!)

Once you are done writing and editing your personal statement, you should proofread it for typos and grammatical errors. Then after that, you should do it one more time, just to be sure. You may even want to have someone else take a look at it because fresh eyes can sometimes see things that you can't. 

 Your personal statement is a reflection of you. So, you want to make sure that it shows your ability to communicate effectively and pay attention to the details. If you submit a personal statement full of typos, the residency directors will assume that you either did not care enough to double-check your work or that you did not notice those typos. Either way, it is not a good look, and it could cost you your spot in a residency program. 

6. Do seek help 

You should consider getting help from professionals if you are unsure how to write your personal statement or if you have written one but you are not sure it is up to par.  Residency match services  can review your personal statement and give you personalized feedback that will help you make it more informative and cogent. 

Furthermore, they can also help you with the preparation of other tricky residency application components like a   residency letter of intent , residency CV, and other Supplemental application documents.

Unless the instructions specifically state otherwise, your family medicine residency personal statement should not be longer than 850 words. 

In short: Yes! Your residency personal statement not only humanizes your application, but it gives you a chance to talk to the program directors and tell them why you want to pursue your chosen specialty. A strong personal statement can significantly improve your chances of matching to your dream program. 

You can start writing early, so that you have time to brainstorm thoroughly and write carefully. Using anecdotes and specific examples in your essay is also a great way to stand out while showing the reader why you are the ideal candidate. 

We recommend that you start writing at least six weeks before you have to submit your essay. 

You can discuss red flags only if you haven't already addressed them in a different application component and they are relevant to your statement. If you do address any areas of concern, make sure you take ownership of the problem and explain how you learned and grew from your mistakes.

Family medicine is not one of the most competitive residencies , but you need to remember that all residencies are competitive to an extent so you will still need a strong application that stands out to increase your chances of getting a match. 

Family medicine is definitely an  IMG-friendly residency program , and also one of the most accessible ones when it comes to competition. So if you write a compelling statement and meet the program’s requirements, you have a pretty good chance of getting into a program. 

Absolutely! While they cannot write the essay for you, they can help you brainstorm, provide tips and strategies to help you write, and help you through the editing process to make sure you are writing an outstanding residency personal statement. 

Want more free tips? Subscribe to our channels for more free and useful content!

Apple Podcasts

Like our blog? Write for us ! >>

Have a question ask our admissions experts below and we'll answer your questions.

This is really helpful thanks! Do you have a similar thing for Pediatrics?

BeMo Academic Consulting

Hello Selma! We sure do! Please check it out here .

Get Started Now

Talk to one of our admissions experts

Our site uses cookies. By using our website, you agree with our cookie policy .

FREE Training Webinar: 

How to make your residency application stand out, (and avoid the top 5 reasons most applicants don't match their top choice program).

family medicine personal statement samples

UW Department of Family Medicine

  • Patient Care
  • School of Medicine
  • Medical Student Education
  • Residency Network
  • Sports Medicine
  • Palliative Care
  • Osher Center
  • MAKE A GIFT
  • Applying to Residency

Impressing: Personal Statement

The best personal statements are memorable. They paint a picture in the mind of the reader and tell a story about who you are, how you got here, and where you want to go. The personal statement is vitally important because it is frequently used to help determine who gets interviewed and ranked. Overarching theme: Look over your CV and think about the experiences before and during medical school that inform what kind of family physicians you will become. Often there is a common thread that holds together even the most disparate of experiences – this common thread is usually one of your core values as a person. Identify this theme and write your personal statement so the reader could easily verbalize this theme in one sentence after reading your statement. Experiences to highlight: Use your experiences to give programs an idea of who you are. Be specific – talking about the aspects of care that you like in Family Medicine is good, but it’s even better when programs can see how your personal experiences reinforce aspects of family medicine that resonate with you as a person. It’s okay to include patient vignettes and talk about your accomplishments, but be sure to relate it back to yourself. How did the experience impact you? What did you learn about yourself? How will the experience make you a better family physician? What about the experience demonstrates your commitment to the discipline of family medicine, your ability to work with others, your ability to work with patients? Choose one experience and tell a story. This is a good way to open your statement, to develop your theme and make it memorable. Commitment to specialty: Talk about why you are choosing family medicine. Programs want to know why your’e attracted to a career in family medicine. What experiences convince you that this is the right field for you? Strengths that you bring: What do you bring to a program? What are you naturally good at? What specific skills do you have that will serve you well in residency? Future plans/what you are looking for in a residency program: At the end of this long road of school and training, what kind of work do you see yourself doing? What types of training do you want during residency to be able to accomplish this goal? Organize your statement: There are many ways to organize your statement to get these points across. One common way of organizing the personal statement is a three paragraph form reminiscent of those essays you had to write in high school. To use this approach the first paragraph tells a story to open the theme, the second paragraph fleshes out other experiences that highlight the them and discuss your commitment to family medicine, and the third paragraph reviews your strengths and future plans/training desires. However, this is a personal statement and you are free to write and organize it as you desire. Do:

  • Write in complete sentences.
  • Use the active voice.
  • Make your writing interesting – use a thesaurus and vary sentence length and structure.
  • Have other people read your personal statement and give feedback.
  • Give yourself plenty of time to work on your statement and revise it based on feedback.

Don’t:

  • Rehash your CV or write an autobiography.
  • Use abbreviations – spell things out.
  • Violate HIPPA.
  • Start every sentence with an “I.”
  • Make it longer than one page, single spaced, 12 point font.
  • Have spelling or grammatical errors.
  • Write a statement that could be used for several different specialties (i.e. one that talks about wanting a primary care career but not specifically family medicine). If you are still deciding on a specialty and applying to different fields, write two different statements.

family medicine personal statement samples

  • Education Home
  • Medical Education Technology Support
  • Graduate Medical Education
  • Medical Scientist Training Program
  • Public Health Sciences Program
  • Continuing Medical Education
  • Clinical Performance Education Center
  • Center for Excellence in Education
  • Research Home
  • Biochemistry & Molecular Genetics
  • Biomedical Engineering
  • Cell Biology
  • Genome Sciences
  • Microbiology, Immunology, & Cancer Biology (MIC)
  • Molecular Physiology & Biological Physics
  • Neuroscience
  • Pharmacology
  • Public Health Sciences
  • Office for Research
  • Clinical Research
  • Clinical Trials Office
  • Funding Opportunities
  • Grants & Contracts
  • Research Faculty Directory
  • Cancer Center
  • Cardiovascular Research Center
  • Carter Immunology Center
  • Center for Behavioral Health & Technology
  • Center for Brain Immunology & Glia
  • Center for Diabetes Technology
  • Center for Immunity, Inflammation & Regenerative Medicine
  • Center for Membrane & Cell Physiology
  • Center for Research in Reproduction
  • Myles H. Thaler Center for AIDS & Human Retrovirus Research
  • Child Health Research Center (Pediatrics)
  • Division of Perceptual Studies
  • Research News: The Making of Medicine
  • Core Facilities
  • Virginia Research Resources Consortium
  • Center for Advanced Vision Science
  • Charles O. Strickler Transplant Center
  • Keck Center for Cellular Imaging
  • Institute of Law, Psychiatry & Public Policy
  • Translational Health Research Institute of Virginia
  • Clinical Home
  • Anesthesiology
  • Dermatology
  • Emergency Medicine
  • Family Medicine
  • Neurosurgery
  • Obstetrics & Gynecology
  • Ophthalmology
  • Orthopaedic Surgery
  • Otolaryngology
  • Physical Medicine & Rehabilitation
  • Plastic Surgery, Maxillofacial, & Oral Health
  • Psychiatry & Neurobehavioral Sciences
  • Radiation Oncology
  • Radiology & Medical Imaging
  • UVA Health: Patient Care
  • Diversity Home
  • Diversity Overview
  • Student Resources
  • GME Trainee Resources
  • Faculty Resources
  • Community Resources
  • Writing a Personal Statement

Personal Statement

The personal statement is an important component of your application. While it’s impossible to know the exact “weight” that a specific examiner will give to this aspect of your application, best estimates range from 5-25%. This is less than the relative contribution of your grades or Boards scores but a sizable chunk none-the-less.

Like many writing samples, there is no “perfect” way to go about writing your personal statement. It is a unique opportunity to let “you” shine through. This is in fact, as the name implies the most “personal” aspect of your application. It gives the reviewer an opportunity to begin to understand you as a person and the aspects of medicine that appeal to you. Only the interview (if you get one) provides a greater opportunity to highlight your personal qualifications.

The following is a list of tips and advice to consider that will help you put your best foot forward with this aspect of your application:

  • Be positive. This is perhaps the most important piece of advice. Reviewers don’t want to hear you rant on about how this specialty or that specialty is not as important or meaningful as family medicine. Share the positive aspects of whatever the topic is that you are addressing
  • Decide what you want to highlight before you start writing. Ask yourself how this will compliment or reinforce the other aspects of your application. Don’t go on a stream of consciousness bender
  • This is not your opportunity to confess all the misgivings, second thoughts, and deliberations you have had over the years about going into medicine or choosing a specialty. If you really think the process of how you made the decision is critical, check with your advisor to make sure you are correct.
  • Be authentic and write from your heart. If you covet continuity with your patients it’s fine to share this in your statement. Don’t worry too much about sounding cliché, unless of course, you are. Back up your statements with brief examples or anecdotes to illustrate your point to help avoid slipping into truly cliché prose.
  • Don’t try to do too much. You need not convey every last thought about why it is you think family medicine is the best specialty in the world. A few, well-crafted and smartly supported concepts often make for a powerful statement.
  • Patient stories are fine if they illustrate a specific point. We want your story, not someone else’s. Obviously, be careful about potentially identifying statements or if appropriate (needed) as for permission.
  • You must have reviewers to give you feedback. Start with a close friend or family member who knows you and can review your statement to make sure your “voice” is reflected in the statement. Have someone read it purely from a proofreading standpoint. Typos cannot be tolerated. Share your final draft with your Residency Advisor for feedback.
  • Our department has 10 copies of Strunk and White, Elements of Style, a text that can be very helpful if it has been a while since you have attempted this type of writing. If you are interested, come get one from our Student Programs Coordinator, Grace Bernier.
  • Talk with your advisor before striking out to use your personal statement to explain some form of irregularity or “problem” in your application. The personal statement may or may not be the place to do so. You can also check with your College Dean for advice.
  • Use the space that is given to you. Not necessarily every last character line but a personal statement that uses only half the allotted space is a red flag.
  • Starting thinking about your statement early. Consider creating a folder (either virtual or real) that you can periodically put some ideas that you are considering incorporating into your statement. These can be concepts, short phrases, actual sentences, paragraphs, patient stories or any other bits of information that will help you to craft your final product. Try and avoid the panic of a rushed personal statement. Like spaghetti sauce, a personal statement that is allowed to “simmer” over days to weeks inevitably results in a more effective message.
  • Clinical Facilities
  • Signing up for Fourth-Year Electives
  • Choosing a Faculty Residency Advisor
  • Asking for Letters of Recommendation
  • Picking a Residency Program
  • Top 15 Residency Interviewing Tips
  • Third-Year Family Medicine Clerkship
  • Fourth Year Electives
  • Family Medicine Activities & Opportunities
  • Faculty and Staff
  • Interviewing Our Faculty
  • Where Are Our Graduates?
  • Class of 2025
  • Class of 2026
  • Class of 2027
  • Community Outreach/Engagement
  • Curriculum Highlights
  • Resident Wellness
  • Residents’ Salaries and Benefits
  • Where We Work
  • Where We Live
  • Diversity and Inclusion
  • Interviewing with UVA Family Medicine
  • Program Director
  • Former Fellows and Positions
  • Research and Teaching During Fellowship
  • Salary and Benefits
  • Life in Charlottesville
  • Application
  • Events and Meetings
  • Patient Resources
  • Research and Scholarship
  • Research Pilot Grant Program

family medicine personal statement samples

  • Our features
  • Our Services
  • How We Work
  • Testimonials

TOP SAMPLES FOR A QUALITY FAMILY MEDICINE RESIDENCY PERSONAL STATEMENT

Get inspired with the best samples and get writing help with your own family medicine residency personal statement.

Creating Family Medicine Personal Statement From Scratch

It’s been a while since a personal statement residency family medicine became an essential part of the application process. In this writing piece, applicants should showcase their strengths and best qualities to assure the institution officials they will perfectly match the program.

However, sometimes writing an engaging and focused family medicine personal statement from scratch may be complicated. Especially if there are too many requirements set by the admission board. And neglecting quality is not an option because such a document is a core application component, and family medicine residency officials review it attentively. That’s why we’ve prepared some practical tips on achieving the highest quality, as well as ready-made personal statement examples medicine that can help you understand what this task requires.

Get Inspired With the Best Family Medicine Personal Statement Examples

Exploring dozens of writing guides, tips & tutorials won’t effectively compose good text until you don’t find family medicine personal statement examples to visualize them. Having a well-composed sample lets you see how the information is structured, what methods and techniques authors use to describe themselves, what accents to make, and how to place them within personal stories. Just have a look at these samples.

family medicine personal statement

Whether it’s an application for a hospitalist fellowship family medicine , or any other one, each document should be unique. Even if the samples are related to one topic, how authors self-present themselves is remarkable.

Once you read a pro-made sample residency personal statement, you will be able to see how each point is managed in writing and how experts work on building structure and formatting with attention to detail.

  • Most authors separate the text into small paragraphs when describing personal motivation, achievements, etc.
  • They carefully add information to support your opinion and make your family medicine residency personal statement more valuable.
  • Pros choose only important facts dedicated to the central idea of your family medicine application doc.
  • They don’t make parts too big and monitor the coherence of the narrative so that the text is not only readable but also consistent and logical.

And these are just a few ideas you may get from a family medicine personal statement example. Of course, you cannot copy from it and use someone’s saamples without customization. But such insights can be beneficial to apply to your own writing.

Check Multiple Medical Samples at Residencypersonalstatements.net

The wide sample selection allows you to find the one that fits your needs. An excellent personal statement residency example composed by one of our writers shows what good application writing should look like. Each expert who works on family medicine samples has relevant experience and understanding of representing the applicant’s personality.

outstanding family medicine personal statement samples

Our medical residency personal statement samples are done according to the admission board requirements, including format, style, word count, etc. Papers are written to represent individuals as perfect candidates for the chosen family medicine program. When another sample is ordered, our experts work on it from scratch, writing it according to the institution’s requirements.

How Good Medicine Personal Statement Examples Can Help

Family medicine programs are quite competitive, so you may go beyond your limits to impress the admission board with your personal statement. At the same time, completed medicine personal statement examples may help you understand how the winning document should look like and give you some ideas about creating the same one yourself.

family medicine personal statement sample tips

The Size Matters

The first you should pay attention to when writing personal statement for family medicine residency is its size. You will never find a sample that goes beyond the word count. 450-600 words – it’s the perfect size of the personal statement you should strive for.

How to Make Your Personal Statement Stand Out?

Next, we’ll figure out more details that must be in your family medicine residency personal statement. However, the personal touches could make it really memorable. Pay attention to how sample authors use customers’ clinical and life cases, creating entertaining anecdotes that catch readers’ attention.

What About Addressing Red Flags?

Your family medicine residency personal statement is a great place not only for self-presentation but also to address some gaps or red flags you may have – e.g., professionalism issues or a too long time after graduation. In a fellowship application, you should be honest and don’t hide anything, but there are no restrictions for ways of presenting details. Just look how our best personal statement samples residency authors turned obvious disadvantages into advantages and ways to overcome difficulties.

Get Started as Early as Possible

Thinking about when to start writing a personal statement internal medicine residency , we can say one thing – don’t waste time, and get started as early as possible. Leaving everything until the last moment affects the quality, as you may not have enough time to prepare for writing or carefully proofread. If you did everything the last night, how would the personal statement for family medicine look? You know the answer.

Things in a Personal Statement Samples Residency to Consider

Each application document should be unique as it represents you as a family medicine specialist and describes your candidacy from the best angles. Plagiarism is unacceptable, but using personal statement examples for medicine allows you to see what general points must be added.

  • Your motivation

What made you choose this path? Why are you passionate about becoming a family medicine specialist? Maybe someone or something inspired you to investigate this field – mention this in a personal statement for pediatric residency programs .

  • A reason to choose a particular residency

What are your expectations from a chosen family medicine residency, and what skills and knowledge you are willing to get? Also, explain how you will overcome obstacles and challenges to succeed in the future.

  • Relevant skills and abilities

Why institution officials should consider you as a great candidate for their residency. Your personal statement for family medicine residency is what may convince the admission board to choose you as their #1 applicant. Show your uniqueness and demonstrate how passionate you are about family medicine. Also, mention your knowledge of the residency opportunities and faculty culture and explain why it is a great place to realize your prospects.

get a family medicine personal statement sample

Carefully edit and proofread the text when it’s done to ensure you don’t have any grammatical or contextual errors, typos, or formatting mistakes. Give your document to a professional residency personal statement writer , so a fresh look at the writing may spot something you may miss.

What You Don’t Find in Medical Residency Personal Statement Samples

As well as essential things, some points should not be seen in the medical residency personal statement samples. Now, we will focus on the typical issues. So, when writing residency applications, forget about such things:

  • Copypasting your CV or writing an autobiography. Your task is to write an attractive and catchy essay introducing yourself to the institution officials. No one is interested in reading the list of your job history or stories from your childhood.
  • Overdetail. Each sample family medicine personal statement has limited space, so there’s no need to count all your school achievements or class ratings. Focus on mentioning cases that improved your professional qualities and made you better in the selected field.
  • Surpassing the word count. Once the medical residency personal statement family medicine has strict requirements, stick to them. That’s why you may need more than one draft to ensure you don’t have unnecessary information in the text.

Get Help With Your Family Medicine Residency Personal Statement Example

Besides sharing guidelines and tips to help future medical program members, we can offer one more option. Our talented authors are ready to help you with your own family medicine residency personal statement example, creating an outstanding one that presents your candidacy in the best way. Having solid experience in application docs preparation, they know how to compose the text in the needed format following all the admission requirements. Be sure all your top traits will be emphasized and the program’s features are addressed.

Forget about struggling with family medicine personal statements and let the professionals do all the work instead of you!

Writing a Personal Statement for Residency Application

Personal statements are an essential, required part of applying to residency. Residency programs screen thousands of applications every cycle and read many hundreds of these statements in the process. You should aim to write an interesting statement that showcases your personality as well as your achievements. Perhaps most importantly, you will need to skillfully articulate the reasons for your interest in family medicine and the particular program you're applying to.

How to Write a Great Personal Statement

A great personal statement sets itself apart from a good personal statement in several ways.

  • First, it includes a level of specificity that shows your motivations and interests are authentic. For example, when conveying why you want to match into family medicine, show awareness of the exciting developments in the specialty, or describe your experience with or knowledge of topics like population health management, care coordination, and the social determinants of health.
  • Feel free to highlight items in your CV if they help remind your reader of the experiences you’ve had that prepared you for the position. This is your opportunity to expand upon activities that are just listed in the CV but deserve to be described so your reader can appreciate the breadth and depth of your involvement in them. It should not be another comprehensive list of your activities, but rather should refer to activities that are listed in detail on the CV.
  • The personal statement is also an appropriate place to address anything that may be ambiguous on your CV. In particular, you should address any nontraditional path you’ve taken through medical school, such as time off or an altered curricular journey. It is better to address these than to leave a program wondering. If you write about academic or personal challenges that you faced during medical school, make a positive impression by focusing on what you've learned from those experiences and how they brought you to where you are now. 

You may choose to relate significant personal experiences, but do so only if they are relevant to your candidacy for the position.

Sharpen Your Writing Skills 

The importance of good writing in a personal statement cannot be overemphasized. Unfortunately, not only are good writing skills allowed to deteriorate during medical school, but in some sense, they also are deliberately undermined in the interest of learning to write concise histories and physicals. For the moment, forget everything you know about writing histories and physicals. While preparing your personal statement:

  • Avoid abbreviations.
  • Avoid repetitive sentence structure.
  • Avoid using jargon. If there is a shorter, simpler, less pretentious way of putting it, use it.
  • Don't assume your reader knows the acronyms you use. As a courtesy, spell everything out.
  • Use a dictionary and spell check. 
  • Use a thesaurus. Variety in the written language can add interest, but don't get carried away.
  • Write in complete sentences.

If you need a crash course in good writing, read  The Elements of Style ,  Fourth Edition  by Strunk and White. If you have friends or relatives with writing or editing skills, enlist their help. Student organizations at your school may host personal statement clinics, or your school may offer review services. Many student, medical, and specialty societies, local and national, may offer personal statement reviews or workshops.

Even if you're a great writer and feel confident about your application, you should ask trusted advisors, mentors, and friends to critique your personal statement (and your CV! ). They can help you make your statement as flawless as possible by giving you feedback about areas that might have been unclear or things that should be added.

Don't cross the line

Your personal statement should remain an original composition, even as you seek input and advice. Retain your voice as you refine your writing and don't ever plagiarize. Be aware of other ethical lines you shouldn't cross as well, for example, don't use vague references that would allow for the reader to misinterpret the nature of your experience, and don't take full credit for a project if others worked on it with you.

Copyright © 2024 American Academy of Family Physicians. All Rights Reserved.

  • Text or Call Us 917-994-0765

family medicine personal statement samples

Residency Personal Statement Examples

Your personal statement for residency is your chance to shine and demonstrate to programs why they should choose you! Begin drafting it early so you can have those close to you review it to ensure it truly reflects who you are and what you offer. We hope the following residency personal statement examples will inspire your creativity.

Please note that these are final drafts. It took multiple rounds of revisions to reach the draft you are about to read. 

family medicine personal statement samples

Family Medicine Personal Statement

OBGYN Personal Statement

FAMILY MEDICINE RESIDENCY PERSONAL STATEMENT EXAMPLE

“Yup, this is what I want to do!” Is what 10-year old Yenchi thought after her well child visit. I became interested in family medicine simply because I wanted to be like my family medicine physician. She became my career inspiration. From my experiences before and after medical school, I learned that family medicine is a flexible and rewarding specialty, which made it more attractive. To make it a rewarding career, I learned that a family physician should have the ability to create a rapport with patients, be organized, and take the initiative. With these abilities, I can better serve my patients and community as a family medicine physician.    

During my rural family medicine rotation, I had a patient named “Rose”. She was well known as a stoic and hesitant person. During our conversation, her answers were short, but her eyes twitched when I brought up her husband’s recent passing and asked how she was faring. She immediately confirmed that she was okay. Sensing that she wanted me to move on from the subject, I asked for her medication list. While Rose was fishing out an aged piece of paper, she gasped, teared up, and apologized for her outburst. She saw her husband’s medication list which brought back a flood of memories to her. I held a tissue box up while she dabbed her tears and she shared her stories about her husband, his battle with cancer, and her transition from wife to long-term caretaker. Through showing patience and empathy, I was able to build a rapport with Rose which allowed her to feel comfortable to show that type of vulnerability with me. This type of connection allowed me to really understand her experience and in the end, allowed me to provide the support that she needed. Through my ability to build a rapport with patients, I would be able to create and maintain longitudinal relationships and care for my patients as a family physician. 

In order to maintain longitudinal relationships with patients, family physicians must stay organized as they focus on many patients and their care. I had the opportunity to exercise my organizational skills after initiating a research project that focuses on the effectiveness of a summer education pipeline program, Project Health Education Advocacy Reflection and Training (HEART). This was designed for high school students in the Mississippi Delta region to explore healthcare career options to meet the increasing need for healthcare workers in this region. I focused on reading other pipeline project articles to produce Project HEART-specific surveys along with creating consent forms and analyzing data. Because there were many components that I had to juggle on my own, I quickly fine-tuned my organizational skills to meet expectations. I can succeed as a resident while utilizing this skill to finish multiple tasks and care for my patients. Furthermore, staying organized will allow me to assume other roles as an attending physician in my community.      

Family physicians have the opportunity to be leaders in their community and to take the initiative to help better their community. I have taken many leadership positions since college. One of those experiences included being Vice President in the Family Medicine Registered Student Organization (RSO). One of my goals as Vice President was to establish the RSO as a local chapter under the American College of Osteopathic Family Physicians (ACOFP). As a local chapter, the RSO can receive the necessary support for future RSO members to pursue any endeavors such as supporting students, who are interested in rural medicine, attend rural medicine interest field trips around Arkansas. My skill in taking the initiative will help contribute to improving the quality of lives in any community I serve as a future family medicine physician.

I fell in love with family medicine because it is unabashedly flexible so anyone can shape their job to a fulfilling career. However, my experiences throughout medical school helped sustain my love for family medicine. Through these experiences, I learned how to successfully connect with patients to establish longitudinal relationships with patients. In order to do so, I equipped my abilities to stay organized and take the initiative to assist me in providing quality care for my patients. With these abilities, I desire to be the type of family medicine physician who is dedicated to bettering the quality of   my patients’ lives, from young to old. 

OBGYN Residency personal statement Example

“Get back on D!” My coach yelled as our opponents stole the ball. I quickly pivoted and began sprinting, feeling the beads of sweat fly off my cheeks. My teammate was beside me pointing to an opponent. She was communicating who she was guarding, so I did the same. Our team began to defend as one cohesive unit, patiently awaiting our next opportunity to score. Through my years of competitive basketball in high school and college, I developed qualities of discipline, adaptability, communication, time management, and teamwork. These are skills that I cultivated each day on the court. With 25 hours of college basketball every week, I learned how to prioritize and balance my education, my training, and my personal life. These qualities have prepared me for a residency in Obstetrics and Gynecology.

With my background as an athlete, I knew I wanted to join a field that had variable moments of high stress and excitement. I further discovered my love for this environment during my 4-years as an emergency room scribe. My experiences in the emergency department led me to desire a field that allowed me to use my hands and perform procedures. My interest in women’s health began during my second year of medical school when I taught a course about women’s health at a home for women recovering from drug addiction. During my OB/GYN rotation, I discovered that I am especially drawn to opportunities for continuity of care, discussions about menopause, counseling about birth control, and creating a safe space to discuss sensitive topics. I found it empowering to engage in open conversations about women’s health, and I strive to similarly empower my patients by building the trusting, open, and comfortable space that is unique and essential to this specialty.

Later in my training, during my MFM rotation, I was reviewing the glucose log of one of my pregnant patients with type 1 diabetes, which showed poorly controlled sugars. This had been noted as a trend for this patient in the past. I interviewed the patient about her log, diet, and insulin usage, and initially, she was guarded and closed off. As I probed more and we began to explain the danger of poor glucose management for her baby, the patient became tearful. She quietly revealed that she had not been eating full meals because she had run out of food stamps and admitted to having trouble feeding herself and her two children. Thus, it was a challenge for her to take her insulin as prescribed. We discussed that she did not need to feel ashamed of her food insecurity. I inquired about her access to local food pantries, community gardens, and found her resources to apply for WIC. I felt confident in my ability to discuss these challenges and help find resources in Cincinnati to address her needs because outside of the clinical setting I co-founded the X. With this student group, I work with local organizations to distribute fresh produce to food insecure neighborhoods and help arrange opportunities for medical students to get involved in education and efforts that target food insecurity. My service has made me aware of how our role as physicians is not only to prescribe and educate our patients on insulin, but to address social determinants of health as well. Experiences like these have driven me to continue combating food and housing insecurity through a service and advocacy elective during my fourth year of medical school.

My experiences on the basketball court, serving my community, and working clinically have prepared me for a residency in OB/GYN. I am equipped with the fortitude to persevere in the face of challenges, adapt to quickly evolving circumstances, and provide the best possible care to my patients. During residency, I look forward to applying skills of resiliency and teamwork as I learn alongside my future colleagues and hope to continue understanding how to treat my patients from all angles. My goal is to become a resident who is coachable, empathic, and an exceptional team member.

More residency personal statement examples coming soon! 

Do you need help with your residency application(s)? Our team of physicians are ready to help you match.

  • 917-994-0765
  • [email protected]
  • Privacy Policy
  • Terms and Conditions
  • Medical School Admissions
  • Residency Application
  • Services FAQs
  • The PreMed App
  • MCAT Vitals
  • Meet Our Team
  • Testimonials
  • Join the Team

© 2024 Motivate MD

family medicine personal statement samples

  • All-In-One Packages
  • Personal Statement Editing
  • Essay Brainstorming Session
  • Activities Section Editing
  • Secondary Essays Editing
  • Admissions Consulting
  • Interview Preparation
  • CASPer Test Prep
  • ERAS Common Application Editing
  • Interview Prep
  • Essay Editing
  • Virtual Shadowing
  • MCAT Question of the Day
  • List of Medical Schools
  • Medical School Map
  • Tools for Applying to Medical School
  • Medical School Application Timeline
  • 2024 Medical School Personal Statement Examples
  • 2024 Medical School Secondary Essays Examples
  • FAQs about Medical School Interviews
  • Medical School Interview Common Questions & Answers
  • Motivate MD Team
  • Free Consultation

family medicine personal statement samples

20+ Residency Personal Statement Examples

family medicine personal statement samples

I hope you enjoy reading this blog post.

If you want our team to help you with your Residency Application,  click here .

Your personal statement is an opportunity to tell your story and journey to residency program directors! It’s your moment to shine and make program directors eager to meet you by presenting a compelling narrative that distinguishes you from the crowd.

In this blog, we provide you with a collection of outstanding personal statement examples from diverse specialties that you can use as references when writing your own personal statement for your residency application! 

We also have detailed guides on how to write your personal statement , how to complete your ERAS application , and 200+ residency interview questions.

And now, let’s get started with the residency personal statement examples:

Residency Personal Statement Example #1:

Internal medicine | the basketball player.

A coach’s instructions, two team chants, followed by the blare of a whistle, opened and closed basketball practice every day. With repetition, my teammates and I strove for perfection to build a skill set that could be recalled when it mattered most. To love the sport of basketball is to love the grind. During my internal medicine rotation, I witnessed similar devotion by attending physicians and residents. Determination to master the foundation of medicine while engaging in a cohesive multidisciplinary team is what resonated deeply with me, and greatly influenced my choice to become an internist.

My passionate desire to become a physician first stemmed from when my grandfather was diagnosed with atrial fibrillation and, later on, heart failure. Initially perplexed by the complexity of his diagnosis, I spent hours researching congestive heart failure, determined to find ways to increase his time with us. Being my grandfather’s primary caretaker towards the end of his life instilled the notion of service and fueled my passion for helping others through this career path in medicine.

During my third-year internal medicine rotation, one of my first patients was a 65-year-old female who was diagnosed with pancreatic cancer and began crying as my attending delivered her prognosis. I talked to her every day, trying to make her feel better. While nothing could completely change her affect, she seemed to appreciate my company. As I reflected on her case, I realized how much I enjoyed getting to know my patients and connecting with them personally, in addition to understanding the complex pathology that plagued them. Several similar experiences on my internal medicine rotation drew me to this specialty which offers a holistic approach and appeals to my innate desire to understand how things work. Internal medicine requires one to understand the interactions between the different systems to diagnose and treat a patient effectively. Additionally, I enjoy the acutely evolving nature of this field and the endless fellowship opportunities available upon completion of my residency training.

My passion for internal medicine led me to start the Internal Medicine Interest Group at our school. Listening to the experiences of different internists further solidified my resolve. Seeing the inspiration within the eyes of the younger medical students as our guests talked about this specialty made me realize the value of role models and generational teaching. This was a source of inspiration for me to pursue a career that not only allows me to take excellent care of my patients, but also teach the next generation of doctors on how to do the same. Being the president of this interest group and the point guard for Duke University’s basketball team, I gained invaluable insight as to how my past experiences shaped my ability to do better in the future, so that my team could achieve lofty goals. This awareness will prove to be paramount in the hospital when serving as an internal medicine physician.

As I enter my fourth year of medical school, I realize how similar medicine and basketball are. The teamwork, which unifies everyone towards a similar goal, the perseverance and long hours required to master the profession, and, arguably the most important, the confidence and trust you build between the team and the people relying on its performance, are critical to medicine and sport alike. Just as I was a trusted member of my basketball team who always put the team’s interest above mine to ensure our success, I am determined to serve as an integral part of the medical team and will do my best towards becoming an excellent clinician while training at your residency program.

family medicine personal statement samples

Residency Personal Statement Example #2:

General surgery | the role model (with commentary).

“Medicine is not a job, it is a way of life.” As the son of a cardiothoracic surgeon, my father’s mantra constantly echoed in my mind. I was raised in an environment where sacrifice and duty were familiar concepts from a young age. While my father did his best to balance work and family life, there were countless occasions when he had to prioritize his patients and commitments over personal events. Seeing his dedication and the impact he had on the lives of his patients, residents, and staff left an indelible impression on me.

After four challenging years studying biomedical engineering in undergrad, I was fortunate to be accepted to the University of Miami’s School of Medicine. While I was genuinely fascinated with almost every discipline of medicine, I had a particular interest in surgery. To give myself time to mature and explore this path further, I elected to take a research year after my second year of medical school and was able to secure a position in the laboratory of Dr. Seth Reigns, director of the Miami Transplant Institute. In the lab, I was tasked with characterizing Regulatory CAR-T cell populations in nonhuman primates. Excitingly, we found that two infusions of Regulatory CAR-T cells are able to prolong renal allograft survival in the absence of traditional immunosuppression. From a clinical perspective, witnessing the transformative impact of liver transplantation on critically ill patients was awe-inspiring. The chance to participate in donor procurements and witness the miraculous recoveries of patients postoperatively further solidified my resolve. Dr. Reigns, a true life-giver, provided me with a profound appreciation for the field of transplant surgery.

During my research year, I had the opportunity to hone my research skills and make significant contributions. However, it was my immersive experience as a third-year clerk on the trauma service that solidified my desire to pursue a career in surgery. Witnessing the remarkable expertise of the chief residents and attending surgeons in swiftly assessing and diagnosing patients amidst the chaos of the trauma bay, where vital information was often scarce, left me mesmerized. The urgency with which they inserted chest tubes and promptly performed emergent exploratory laparotomies was nothing short of exhilarating and profoundly inspiring. Equally fulfilling was the privilege of accompanying these patients throughout their hospitalization, observing their remarkable recovery from being intubated in the intensive care unit to the triumphant moment of their eventual discharge. This comprehensive experience further affirmed my passion for surgical intervention and reinforced my unwavering commitment to becoming a surgeon.

In addition to my research endeavors, I also became involved with Operation SECURE, a nonprofit crisis center in Miami that offers crisis counseling services free of charge. This experience has been humbling and rewarding, particularly as I counsel individuals struggling with alcohol and substance use disorders. Drawing from my background in transplant surgery, I am able to provide a unique perspective on the long-term consequences of addiction. While surgical intervention can address these issues this experience demonstrated the importance of preventative medicine as well.

Looking ahead, my goal is to pursue a residency in general surgery, with the ultimate aim of specializing in abdominal transplant surgery through a fellowship program. I am well aware that the challenges I will face in my training are formidable, but I am constantly reminded of my father’s voice, urging me to approach this as more than just a job—a true lifestyle that demands my unwavering commitment. As I embark on this journey, I am eager to give everything I have to the field of surgery. It is my steadfast dedication to making a profound difference in the lives of patients, the pursuit of knowledge and innovation, and the opportunity to live my dream that fuels my passion for general surgery and the transformative field of transplantation.

Commentary on Residency Personal Statement Example #2

The first paragraph is what will set the tone for the entire personal statement. Ideally, you can open up with an engaging first sentence that will “grab” the reader. In this case, the applicant is providing a quote from her father describing the sacrifices that one must make as a physician. The applicant then sets up her father as a role model and the role this played in her decision to pursue medicine.

Note that often applicants feel the need to be “too creative” in the opening paragraph. A quote from a mentor or influential person or patient is ok, but you don’t have to always include quotes or extremely unusual stories. Further, recognize that some applicants will have more unique or interesting personal experiences than others. Not every applicant is a cancer survivor or has donated an organ to a family member or is the product of a war-torn country. The overall goal of the personal statement is to provide a concise, polished essay demonstrating your motivations for residency. Along the way, you tell your story while highlighting key aspects of your personality and CV.

These next two paragraphs are perhaps the most important. Here the applicant dives into what made her want to become a general surgeon. She talks about her research experiences in a surgical lab and her clinical experiences with her mentor Dr. Reigns. Note that while she is not simply rehashing her CV, she does mention her academic accomplishments and drives key points home. Note that while the applicant elected to open the first paragraph with a quote from her father, she could have also chosen to open with an internal thought or reflection from these clinical experiences with Dr. Reigns (i.e., “I’ll never forget the moment we completed the venous anastomosis and ended ischemia time. Blood began perfusing the pale liver as it pinked up.”)

This paragraph draws on another crucial experience that the applicant had outside of the lab/OR. Remember, you are presenting yourself as a whole person so it is important to mention any other influential experiences (volunteering, service, etc.) that you are particularly proud of. Also, note that while the applicant is serving as a crisis volunteer, she circles back and relates it to her prior experiences above.

The final paragraph is also very critical. Here you should mention your long-term goals. It is ok to be vague and specific at the same time. Finally, you should try to tie things up and if possible, connect them to any comments made in the first paragraph. Here the applicant paraphrases her father’s quote that opens the personal statement. Finally, the applicant affirms their choice for applying to general surgery and provides an optimistic look on their future training.

As a final note remember that the personal statement is just one piece of an entire application. While it is important most applicants do not get an interview based on a personal statement, however, rest assured some applicants do not get an interview based on a poor personal statement. The vast majority of personal statements (~85%) are simply acceptable documents that tell your personal journey while mentioning key aspects of your application. They are well-written, logical, and polished with no grammatical errors. A small portion (less than 5%) are truly incredible literary documents that are beautifully written and tell an incredible story. Still, these personal statements will likely do little in the way of getting you an interview. Finally, the remaining 10% of personal statements are the ones that can have your application dismissed. These personal statements are unpolished, contain grammatical errors, or are trying too hard to fall in the top 5% and come across poorly.

If you are looking for a comprehensive ALL-IN-ONE Application Resource for MATCH® 2025, including ERAS application template, personal statement examples, MSPE samples, LOR examples, and much more, click here .

Residency Personal Statement Example #3:

Internal medicine | the healer.

Growing up in a rural Kenyan village, where my father, a traditional healer, used herbs to alleviate suffering, and my mother, a teacher, instilled in me the value of knowledge, I developed a respect for the healing arts and a commitment to education. This unique upbringing laid the foundation for my medical journey, intertwining traditional practices with a scientific approach. My decision to pursue a career in medicine was cemented when an American medical mission dramatically improved healthcare outcomes in my community, including saving my neighbor’s life from severe malaria. This experience unveiled the stark disparities in healthcare between developed and developing nations and inspired my dream to bridge this gap.

After completing my medical degree at the University of Nairobi, I was accepted into their Masters of Medicine in Internal Medicine (equivalent to residency). During my training, I was exposed to a broad spectrum of diseases rarely seen in more developed nations and recognized the need for advanced training to bring substantial change to healthcare practices in my home country. It was this realization that prompted my journey to the United States, seeking a residency in internal medicine, motivated by the opportunity to learn from and contribute to one of the world’s most advanced healthcare systems.

Passing the USMLE exams was my first challenge, which I approached with diligence and the support of mentors who were instrumental in my academic and personal growth. My scores reflected not only my understanding of medical sciences but also my commitment to achieving my goals.

In the U.S., I secured opportunities to shadow internal medicine physicians in various settings, from city hospitals to rural clinics. These experiences enriched my understanding of the nuances of patient care across different contexts. They underscored the importance of cultural sensitivity, adaptability, and the need for a personal touch in patient interactions—qualities that were greatly appreciated by my peers and supervisors. While I have always been drawn to internal medicine, my experience in the US only served to strengthen this affinity. Given the greater accessibility to healthcare here, I felt that internists could truly change patients’ lives by providing routine health screening and maintenance. The ability to develop lifelong relationships with patients and their families is also an incredible privilege almost uniquely afforded to physicians.

My career aspirations do not stop at becoming a skilled clinician. I am driven by a vision to integrate effective, sustainable medical practices from the U.S. into the Kenyan healthcare landscape. While I remain open to pursuing a fellowship, my goal at present is to become a hospitalist, leveraging my training to tackle prevalent health issues in both the U.S. and Kenya. Though I plan to establish my full-time practice in the US, I look forward to establishing programs in my home country. Through collaborative research and leadership, I aim to develop healthcare models that are both innovative and adaptable to the constraints of resource-limited settings in both countries.

The resilience I have developed through my journey—from a small village to the world stage of U.S. medicine—is a testament to my dedication. I have navigated cultural transitions, educational challenges, and professional milestones with a clear vision and a steadfast heart. With each patient I meet and each case I handle, I am reminded of why I embarked on this path: to be a bridge between worlds, offering my skills where they are most needed and fostering healthcare advancements that are accessible and effective for all. 

If you want a detailed guide on how to write a personal statement and things to include in this important document, check out our other blog here .

family medicine personal statement samples

Residency Personal Statement Example #4:

Pediatrics | the indian img.

My journey into pediatrics was inspired not by chance, but by the profound impact of witnessing a loved one’s struggle with illness during my childhood in Pune, India. My cousin Priya’s battle with severe asthma exposed me to the challenges and triumphs of pediatric care. The dedication of her doctors, who turned her tears into smiles, sparked my resolve to pursue a career where I could deliver similar hope and health to children.

During my medical training at the All India Institute of Medical Sciences, I thrived academically and was actively involved in extracurricular activities that reinforced my passion for pediatrics. As president of the Pediatric Interest Club, I led initiatives such as organizing health camps for underprivileged children and spearheading an asthma awareness campaign in local schools. These experiences not only honed my leadership skills but also deepened my understanding of pediatric health challenges. My efforts were recognized when I received the ‘Best Student in Pediatrics’ award during my final year. Encouraged by my mentor, Dr. Meena Singh, to seek out the most advanced training, I was drawn to the United States for its exemplary integration of evidence-based medicine, cutting-edge research technologies, and innovative clinical practices.

In pursuit of this advanced expertise, I moved to New York two years ago. My clinical observership at New York-Presbyterian Hospital exposed me to diverse pediatric cases and modern treatment modalities, enriching my clinical acumen. Concurrently, I participated in a Columbia University research project investigating the environmental impacts on pediatric asthma, which aligned closely with my interests and previous advocacy work.

One particularly formative experience during my observership involved a young boy with non-verbal autism who presented with acute appendicitis. Navigating his care required not only medical expertise but also profound sensitivity to his unique communication needs. Successfully managing his treatment while ensuring his comfort reaffirmed my commitment to pediatrics, highlighting the importance of tailored and compassionate care.

As I seek to join a pediatric residency program, my goal is to become a skilled pediatrician equipped to handle the complexities of child health. I am especially drawn to pediatric pulmonology, but I remain open to exploring all pediatric subspecialties to build a comprehensive skill set. Beyond residency, I envision working in a rural area in the U.S. where I can make a significant impact on underserved communities. I am also committed to establishing collaborative health initiatives that bridge the gap between advanced care in the U.S. and the needs of pediatric patients in India.

My path from Pune to New York has been a journey of growth, guided by a mission to improve children’s health globally. I am eager to bring my background, clinical insights, and dedication to your program, contributing to and benefiting from a community that champions innovative and empathetic pediatric care. 

Residency Personal Statement Example #5:

Family medicine | the caregiver.

Working alongside the primary care physicians in my medical school in India, I was impressed by my preceptors’ abilities to remember every tiny detail from recommending required vaccinations to establishing complex goals of care. During the final days of my rotation, I was fortunate to see these efforts pay off as patients from the weeks before showed up healthy and happy, ready for the next step in managing their health. That is when I began to share the same instinctual gratification as my preceptor when they coordinated multiple levels of care, informed specialists of updated patients’ status and maintained a healthy physician-patient relationship. Heading into the final year of medical school, I knew that I wanted to become a family doctor.

To gain more experience in the field while engaging with a different patient population, I pursued an elective at Boston University. Under the guidance of Dr. John Smith, I started to further improve my skills in shared decision-making. This involved making the active choice of incorporating more practical approaches to engage patients with their healthcare including tailoring diet recommendations to patient-accessible foods and prioritizing a few issues at once. While I had ingrained some of these techniques already from watching my medical school preceptors, I believe this opportunity at Boston University allowed me to think more proactively to cater to a more diverse patient population. Anecdotally, I believe this has also resulted in higher rates of therapy compliance and follow-up visit attendance.

Soon after returning from my electives, I began to work towards bringing this form of personalized medicine to rural areas in my home country of India. Alongside some of my fellow students, we began a volunteer initiative that involved a more old-school approach of physically making ‘rounds’ of underserved neighborhoods. We developed and adopted standardized screening questionnaires that helped us identify household members who required medical assessment. We would present our findings to a team of physicians who would then help us provide education, interventions, and medications appropriate to each person’s needs and socioeconomic ability. This opened my eyes to the potential of primary care outside of the hospital: health is a continuous element that needs to be addressed daily!

The tailored approach to medicine has also guided my goals in medical research. In my pursuit of academia, I began speaking with some experts in South Asia responsible for the development of practice guidelines after graduation. I realized that one of the biggest disparities in assessing patients is simply not knowing the population-specific normal values of routine labs and examinations. To address this concern, I joined a lab run by Dr. Amir Khan as a post-doctoral research fellow at Mass General Brigham to develop a new set of normal distribution curves for a battery of tests using samples from phenotypically health South Asian individuals living across Greater Boston. Realizing that there is still a wide world of unaddressed issues in primary care has provided additional motivation in my pursuit of a career in family medicine academia. Combined with extensive clinical training that I would acquire from a residency in the United States, I aspire to be a leader in the primary care space, working towards personalized medicine for all.

Overall, I believe that as an academic family medicine physician, one has to make sure the efforts of patients, physicians, and healthcare policymakers are working concertedly towards better healthcare outcomes. My experiences as part of teams providing healthcare to a diverse set of patients, both ethnically and socioeconomically, provide me with a unique perspective that I hope to bring to my future residency program and the world of research and healthcare policy.

Looking for a full ERAS Application Template including samples of Experiences, Education, Geographic Preferences, Publications, and Personal/Biographic Information? Get your FREE ERAS Template here ! 

Residency Personal Statement Example #6:

Emergency medicine | the grocery manager.

Project Open Hand was no ordinary grocery center. It was a bustling, high-energy urban community center for a revolving door of over 200 community members with financial and housing difficulties. As the wellness program director, I managed the center, alongside a team of receptionists, nutritionists, and volunteers who looked to me for guidance. On any given day, I managed conflicts with clients receiving their weekly groceries, communicated with outside organizations to connect clients to resources, and improved organizational processes. Tossed into new situations that would challenge me, I was prepared for any adventure. I saw not only what our team could accomplish, but how I was drawn to vocalize and lead in times of stress and chaos.

Fast-forward to medical school, I did not anticipate that my experiences at Project Open Hand would foreshadow the specialty I would be most drawn to. Throughout medical school, I could see a part of myself in every specialty. I enjoyed connecting with patients in Family Medicine, thinking through complex problems in Internal Medicine, and using my hands for precise procedures in Surgery, but it was in Emergency Medicine where I finally felt right at home. Much like the environment at Project Open Hand, I enjoyed the fast-paced dynamic nature that demanded critical thinking, adaptability, and teamwork. The combination of uncovering clues to help undifferentiated patients and engagement in diverse procedures challenged and excited me. I could never be complacent, as health conditions were constantly changing.

I now want to be a leader in Emergency Medicine and plan to do so in three areas: upholding clinical excellence, contributing to the profession through education, and giving back to underserved communities. First, related to clinical excellence, my research endeavors have taught me the importance of research in guiding clinical practice. For example, through my research on abdominal aortic aneurysms, I learned that gender, along with other factors, can influence the presentation and progression of diseases. Then, while on rotation at Marshall Hospital, I had a patient present with atypical abdominal pain. Utilizing existing clinical knowledge, my team and I diagnosed her with an abdominal aortic aneurysm. Just as in this situation, I aim to apply what has been studied in research to improving diagnosis and treatment plans for patients, especially in the emergency medicine setting where patients are at their most vulnerable.

Second, I want to contribute to the profession through education. While much of emergency medicine treats at the end of a continuously flowing river, I will spend time upstream by training the next generation of emergency medicine physicians. While at American School of Medicine, I was surrounded by women and people of color who taught me to question norms, trust my clinical intuition, and treat patients, not numbers. I value the education I experienced, and I intend to pass this on to young eager residents to train intellectually and culturally competent physicians. I will use technological advances in ultrasound and simulation to guide and improve education. Lastly, I plan to give back to underserved communities by continuing to volunteer to provide education and address community needs. My decision to attend American School of Medicine was fueled by a clear intent to integrate health equity into my clinical practice. With involvement in the community, I am reminded of my motivations.

I seek a residency program with the many qualities of Project Open Hand and my numerous research, service, and clinical experiences. These include being challenged, working in teams committed to a common goal, and committing to excellence and service. In hindsight, Project Open Hand was an opening into the experiences of emergency medicine. I welcome the opportunity to be a leader for such a team again.

If you are applying to the Match and need a detailed guide on how to ace your ERAS application, check out our free ERAS Application Guide here .

Do you want our experienced team to edit your Personal Statement?

family medicine personal statement samples

Residency Personal Statement Example #7:

Pathology | the img pathologist.

Growing up in the early 1990s, I fondly recall accompanying my mother to what would become one of Lebanon’s largest laboratories. With the multiple wars finally behind us, my uncle, a pathologist in New York City, returned to Lebanon to build his pathology lab with my mother’s help. As a child, I watched in awe as they transformed a few rooms into a state-of-the-art facility. During visits to the newly opened lab, my uncle, a board-certified cytopathologist, introduced me to the wonders of cytopathology through a microscope. His confidence and skill in diagnosing cases captivated me, sparking my passion for pathology and shaping my aspiration to excel in this field.

I worked hard during my first two years of medical school, excelling in my classes and even making it on the dean’s honor list in my second year. My keen interest in pathology led me to pursue an observership at George Washington University (GWU) in Washington D.C. during my fourth year. This experience provided me with the opportunity to engage positively with several attending physicians, and I received commendations for my adeptness in making morphological diagnoses. Encouraged by these pathologists to further my career in this field, I followed their advice and have spent the past fifteen months as an Anatomic Pathology resident at the American University of Beirut.

During this period, I managed a diverse array of responsibilities, ranging from working at the grossing bench to examining specimens under the microscope. My public speaking skills have significantly improved through presenting various pathology topics at surgical seminars. As a committed team player, I have mentored new residents, instructing them in the complexities of grossing specimens and managing weekend calls. This mentoring experience has enhanced my leadership and teaching abilities, which I consider essential for any medical professional.

Although I am just fifteen months into my residency, I have already noticed significant improvements in my diagnostic and grossing skills. However, driven by a commitment to continual growth and excellence, I decided to pursue pathology residency in the United States. I have seen firsthand the knowledge and expertise that the training in the United States provides, and I believe that a residency training in the U.S. would give me the education and guidance to become the best all-around surgical pathologist I could be.

Over the past year, I’ve learned that effective communication is crucial for managing a laboratory and that perseverance and versatility are vital for a resident’s development. Participating in double-scoping sessions and signing out cases with attendings has proven essential. Therefore, I am seeking a residency program that not only offers a robust learning environment but also prioritizes educational engagement, where attendings are committed to closely collaborating with residents on case workups and research projects.

My uncle’s achievements have always served as a benchmark for my own aspirations as an emerging pathologist, and I remain committed to the inspirations that launched my career. Dedicated to honing my skills and expanding my expertise, I am confident that I would be a valuable asset to pathology programs that value continuous improvement and dedication in their team members.

family medicine personal statement samples

Residency Personal Statement Example #8:

Orthopedic surgery | the football player.

From an early age, sports have been at the core of my identity. Growing up with parents who were Division I athletes—my mom a volleyball star and my dad a basketball powerhouse—athletics were not just encouraged; they were an expectation. Following in their footsteps, I thrived as a wide receiver in football, eventually playing at the collegiate level for Purdue University. My journey, however, took an unexpected turn when I tore my ACL and MCL during my junior year. This devastating injury abruptly ended my football career but opened a new path that I never anticipated.

My introduction to orthopedic surgery came through my recovery process with Dr. Yang, the surgeon who repaired my knee. Watching Dr. Yang work and observing his precision, dexterity, and the impact he had on athletes’ lives made a significant impression on me. The camaraderie in the training room and the meticulous nature of surgical practice reminded me of the locker room environment I loved. This experience led to a profound shift in my career aspirations. I changed my major from marketing to pre-med, dedicating myself to the rigorous path of becoming an orthopedic surgeon.

During my undergraduate years, I began shadowing Dr. Yang and engaging in clinical research focused on outcomes using cadaveric materials for ligament reconstruction. These experiences solidified my decision to pursue medicine and led to my acceptance at the University of Michigan Medical School.

Medical school was a period of tremendous growth for me, both academically and personally. Outside the classroom, I continued my research in Dr. William Defoe’s laboratory, studying the dynamic interactions between bone cells and the extracellular matrix (ECM). This work was intellectually stimulating and fulfilling, resulting in 15 publications, six of which I authored. My dedication to research was recognized when I received a one-year research fellowship from the Department of Orthopedic Surgery. During this fellowship, I balanced benchtop research with clinical projects in the sports medicine department, presenting my findings at over 50 regional, national, and international conferences.

Beyond academics and research, I found joy and purpose in volunteering as a football coach at St. Basil’s Middle School. For five years, I mentored and coached disadvantaged children, helping them develop not just as athletes, but as individuals. Taking the team to the University of Michigan football games at “The Big House” and organizing bonding activities like bowling and trips to the driving range allowed me to give back to the community and remain connected to the sport I love.

After my research year, I was fortunate to secure sub-internships at the Hospital for Special Surgery, Washington University in St. Louis, and the University of Pennsylvania. These rotations provided me with hands-on experience and reinforced my passion for orthopedic surgery, particularly sports medicine. Importantly, I was able to work closely with the residents, taking 24-hour call shifts, seeing ED consults and afforded graduated responsibilities in the operating room.

Looking to the future, my immediate goal is to match into a robust orthopedic surgery program that will nurture my growth as both a surgeon and a researcher. While I am eager to explore all facets of orthopedic surgery, I have a special interest in sports medicine and plan to pursue a fellowship in this subspecialty. Ultimately, I envision myself practicing at an elite academic medical center where I can operate, conduct research, and teach the next generation of surgeons. I also aspire to serve as a team physician for a professional sports team or a Division I college team, blending my love for sports with my medical career.

Reflecting on my journey, from the devastation of a career-ending injury to the discovery of my true calling in orthopedic surgery, I am grateful for the experiences that have shaped me. Each step, whether on the football field, in the research lab, or in the operating room, has prepared me for the challenges and rewards of a career in orthopedic surgery. I am excited to bring my dedication, resilience, and passion to a residency program that will help me achieve my goals and contribute meaningfully to the field.

Residency Personal Statement Example #9:

Anesthesiology | the immigrant.

“Okay, let’s start masking Violet”, said the pediatric anesthesiologist with whom I was working during my third year. Violet was the sparkly, purple, stuffed cat of our four-year-old patient with sickle-cell disease who was scheduled for a splenectomy. Observing my attending mask our patient’s stuffed cat while I attended to our patient, I was struck by the seamless blend of skill and empathy. Witnessing how my attending effortlessly built trust with a nervous four-year-old and her anxious parents, as we transitioned from playtime to the operating room, left an indelible impression on me. In mere moments, we navigated from moments of joy and laughter to the meticulous administration of anesthesia, followed by insightful discussions on the intricacies of anesthesia physiology. This transformative experience served as the catalyst for my commitment to pursuing a career in anesthesiology.

Growing up as the eldest daughter in a first-generation immigrant family that relocated to the United States when I was twelve, I faced a unique set of challenges. While acclimating to a new culture and education system, I found myself navigating the complexities of language barriers and unfamiliar environments. While my peers focused on building their college resumes, I balanced my academic pursuits with the responsibilities of assisting my parents with my sister’s education, aiding them in job applications, and coordinating doctor’s appointments. Acting as the primary liaison between my family and healthcare providers, I undertook the role of translator during medical consultations, ensuring that my family received the care they needed. These experiences, though daunting at times, served as a driving force behind my aspiration to become a physician, and later, an anesthesiologist. Through these formative experiences, I cultivated independence, resilience, and a deep-seated desire to alleviate the struggles of others. I learned to effectively multitask, maintain composure in high-pressure situations, and swiftly adapt to unforeseen challenges—qualities that are integral to the role of an anesthesiologist.

My interest in anesthesiology eventually led me to undertake several leadership positions during my medical school career. With no dedicated anesthesiology department, I recognized the necessity to forge connections and bridge mentorship gaps not only for myself but also for future students. I eventually formed an anesthesiology interest group at our medical school and served as president ensuring that students had access to mentors within all anesthesia subspecialties. Additionally, I organized several match panels to promote collaboration and to showcase clinical and research avenues on a unified platform. Eventually, I took an even broader role and served as the official delegate for my medical school in the American Society of Anesthesiologists. These experiences highlighted the important role anesthesiologists play as leaders and mentors in the clinical, research, and political landscapes and I hope to continue to expand these skillsets further into residency.

Anesthesiology resonates deeply with my core values and professional aspirations. My commitment to delivering individualized care, mentoring future physicians, and fostering a sense of reassurance and trust during moments of vulnerability align seamlessly with the core principles of this specialty. The breadth of the field and the combination of managing highly complex and ever-changing situations coupled with the ability to practice procedural care makes anesthesiology the perfect career choice for me. Within the field of anesthesiology, my interests lie in pediatric anesthesiology and medical education. I am driven to contribute to a residency program that offers a breadth of clinical experiences, allowing me to encounter a wide spectrum of cases while thriving within a collaborative environment that fosters leadership and mentorship. 

Residency Personal Statement Example #10:

Psychiatry | schizophrenia.

I hear voices! These three words summarized my grandmother’s lifelong suffering. I grew up in an Indian family, accustomed to the tales of old people hearing voices, seeing strange things, and wandering away for months. All this was very commonplace and rarely attended to. In a country plagued with limited access to education and healthcare literacy, mental health disorders were considered a myth. The social stigma precluded discussion of symptoms and provider visits. It was only during my medical schooling that I understood such symptoms to be part of mental illness that affects patients and causes intense distress. As my curiosity was aroused, I found psychiatry to be my true calling.

The opportunity to complete four months of psychiatry rotations during my final year of medical school allowed me to witness and treat psychiatric diseases that I had only known previously as vague symptoms. I remember taking care of identical twins afflicted with schizoaffective disorder stemming from years of extensive emotional and physical abuse by their family. Years of lack of care and social abandonment had resulted in shared hallucinations and delusions, with multiple suicidal attempts. Effectively gaining their trust by validating their concerns enabled me to unveil their self-injurious behavior and suicidality, prompting appropriate management. On subsequent visits, both patients had significant improvement in their symptoms with a more positive outlook and adherence to medications and psychotherapy. Such experiences and many others that followed provided me with an in-depth insight into the contributing factors to mental health disorders and the effectiveness of prompt and adequate treatment in optimal patient recovery.

Since relocating to the United States for a master’s program in clinical psychology at the University of San Diego, California, I have gained clinical and research acumen that has further reinforced my passion for psychiatry. My role as a crisis counselor for the past two years with CalHOPE, California, has provided me with clinical versatility and a profound understanding of patients’ ongoing conflicts. Interacting with hundreds of patients and communities with depression, anxiety, and substance use disorder, has helped me hone my skills as a listener to actively pick up subtle cues and offer a tailored approach to care. Nothing has been more gratifying than witnessing patient improvement with the right treatment.

Currently, I spearhead the research on the psychological effects of drug misuse and addiction in underserved populations along with different strategies to facilitate early diagnosis and intervention. I have learned the skills required to formulate a research question and design a study from an idea to publication and seek to utilize this knowledge to positively impact patient care across the globe. I am passionate about research and working with communities combating drug addiction and mental health stigmatization. Therefore, I seek a residency program that will equip me with the skills to become an excellent psychiatrist and researcher so that I can build therapeutic alliances with diverse patient groups and backgrounds.

My clinical experiences have illuminated that the most admirable physicians are those who cater to the medical and psychological needs of patients from different socioeconomic backgrounds. While my grandmother’s tales of hallucinations served as the fuel that ignited my interest in psychiatry, every experience I went through during my medical journey confirmed that psychiatry is my natural calling. I stand now as an aspirant for this field seeking the requisite training that will enable me to be a beacon of support for communities with mental health disorders and break the barriers of stigmatization and social injustice. 

family medicine personal statement samples

Personal Statement Editing

Our editing includes not only language but also context, structure, and content advising.

family medicine personal statement samples

ERAS Application Editing

The editing goes beyond language and grammar corrections to structure, design, and content based on your personal story and achievement.

family medicine personal statement samples

  • Interview Preparation

The best way to learn something is to do it. That’s why we divide our interview preparation sessions into two parts. Mock Interview + Feedback

family medicine personal statement samples

  • Residency Advising

We are able to provide you with the guidance you need at any step of your journey to make it to your final goal!

Residency Personal Statement Example #11:

Obgyn | the caribbean school.

“Time to close”, said the scrub nurse as she placed the needle driver in my hand, just a few hours after a young female patient had presented to the emergency department at the Sint Maarten Medical Center with vaginal bleeding. Within minutes of her arrival, she was being rolled back to the operating room for a ruptured ectopic pregnancy. As a student rotating on the service, I asked to scrub into the case with the team and was given the opportunity to close at the end of the procedure. That experience was my first exposure to the unique world of obstetrics and gynecology and served as my catalyst for pursuing this specialty.

As a second-generation American immigrant, I had watched both my parents train as physicians in their home country and subsequently re-train in America in their respective specialties. Their sacrifice and dedication towards building a foundation and home for me and my siblings, inspired my work ethic. Their passion and commitment to their patients drew me to the field of medicine.

After persistent efforts, I secured admission into a medical school in the Caribbean. Studying medicine at Sint Maarten, I knew the challenges that awaited along my career path as a physician seeking to integrate into the American residency system. I pursued each opportunity to serve the medical community of Sint Maarten, while advancing my education as I shadowed OBGYN physicians on Saturday mornings, during my free time. I obtained history and examined every patient on the floor prior to them being seen by my attendings. This experience not only improved my clinical knowledge and skills significantly, but also opened my eyes to the diverse needs of the island and its people. Living in Sint Maarten allowed me to witness the effects of low socioeconomic status, lack of resources, and limited medical literacy on the overall health and well-being of a community.

Moving back to the US for my clinical rotations, my passion for women’s health continued to fuel my desire to pursue residency training in OBGYN. Whether it was in the delivery room encouraging a first-time mother or in the clinic counseling a patient with bladder incontinence, I was drawn by the breadth of the practice. During my third year of medical school, I assisted a team of OBGYN residents who were comparing surgical outcomes after laparoscopic versus robotic hysterectomy. This experience showed me the impact that researchers can make on patients’ lives world-wide, and kindled my interest to develop the skillset that propelled an idea to a publication. Presenting our research at the ACOG meeting this past spring allowed me to learn more about the intricacies of OBGYN and engage in meaningful conversations with leaders of the field.

Although that Saturday morning at the Sint Maarten Medical Center sparked my interest in this specialty, it was the culmination of my clinical experiences which affirmed it. I look forward to integrating patient care, clinical skill, and technology in surgical management throughout my residency. By training at an academic center, I hope to continue my contributions to this field as a learner, a teacher, and a leader. The same way my parents inspired my passion and dedication to medicine, I hope to inspire future generations during residency and beyond. 

Residency Personal Statement Example #12:

Diagnostic radiology | the pakistani img.

‘Are you taking a look at that Chest X-ray?’ said an attending pulmonologist behind me as I was staring at a computer that was stuck opening the radiology report. ‘No, sir, but I would love to learn,’ are the few words that started my journey into radiology. Over the rest of the rotation, my attending began to teach me the basics of image interpretation, and by the end, I was able to pick out bits and pieces of pneumonia, atelectasis, and interstitial lung disease on chest x-rays and high-resolution CTs. As an avid fan of mystery novels and languages, I found that piecing together bits and pieces of data gleaned from imaging and conveying these findings to doctors and patients scratched the same itch as when a detective finally has their ‘Eureka!’ moment and presents their case to a jury!

With my newfound passion, I soon began shadowing radiologists at my home institution in Pakistan. Studying the language used in reports and that used by patients, I quickly learned that there was a disconnect that needed to be bridged. Together with Dr. Muhammad Zaheer, I applied my love of languages and puzzle-solving and took the initiative of developing an English-to-Urdu dictionary of words that can help translate common and technical radiological terms into easy-to-understand Urdu words that other physicians can use to convey medical reports. We are currently working on integrating these into an electronic system that can auto-generate a translated report from a radiologist’s interpretation, further reducing the barriers between state-of-the-art medicine and patients who are not necessarily medically literate.

Reducing barriers between the patient and doctor is not enough. Radiology has taught me that the fast-paced and immediately effective nature of work requires the effective application of communication and language skills between the radiologist and other healthcare professionals as well. As an elective student at the University of Pennsylvania, our team of 3 people would read upwards of hundreds of chest X-rays for lung collapse, pneumothorax, pneumoperitoneum, etc. on an average day and would be in constant contact with the relevant physicians to ensure timely care. In addition, we would have to inform several healthcare teams about the status of venous lines, endotracheal tubes, and nasogastric tube placement, which are crucial to continued patient care. During multidisciplinary team meetings, I also came to appreciate the value of imaging during the evaluation of suspicious pulmonary nodules. Conveying these complex concepts in precise, efficient terms further developed my love of radiology, as I could see myself playing a central role as a ‘doctor’s doctor’ in the healthcare system for both acute and chronic conditions.

The intersection between my love of languages, problem-solving, and radiology did not stop there. I learned through my interactions with many brilliant radiology technicians that there is another exciting avenue to connect academic radiology with biomedical engineering, further optimizing patient outcomes. This led me to pursue a post-doctoral research fellowship at Cleveland Clinic under musculoskeletal radiologist Dr. David Johnson. Using basic principles of MRI, we developed new protocols capable of detecting osteoarthritic changes in the knee, allowing early intervention. Using artificial intelligence, we also developed several deep-learning models capable of automatic osteoarthritis feature detection (like synovitis and bone marrow lesions) that can fasten radiologists’ workflow, acting as a side-investigator that alerts them to possibly hidden clues. I believe that I will continue to use my radiology expertise in the future to aid the development of such exciting innovations.

Although it took a malfunctioning computer to introduce me to this specialty, I have come to realize that radiology truly is a cross-section of all my passions. Using the correct phrase and finding a simple solution can make all the difference in guiding a doctor, informing a patient, and shaping research goals. As an aspiring academic radiologist, I aim to continue to connect radiologists with the people whose lives we affect, and I hope to contribute to your program as an inquisitive and collegial resident.

Residency Personal Statement Example #13:

Interventional radiology | the chess player.

My initial experiences with interventional radiology are a great microcosm of all the reasons why this specialty speaks to me. When my mother was suffering from varicose veins, it was an interventional radiologist who was able to ease her pain. Similarly, it was an interventional procedure that embolized a life-threatening bleed for one of my best friends. As a chess enthusiast, that is when I realized that interventional radiology as a specialty functions a lot like the queen piece: it is highly versatile and can be called into action at any time!

Like chess, I soon realized that interventional procedures often allow you to plan several steps, but still require on-the-fly decision making. During my elective time with Dr. John Smith at Medical University, we would spend a fair amount of time planning approaches, instrumentation, anatomy, and ultimately intervention before each procedure using the patient’s medical history and importantly, their imaging. This helped us build a roadmap of what to expect. However, we spent an equal amount of time adjusting to issues discovered in real time such as variant anatomy and unexpected device failures. The culture of learning from each mistake and building expertise in this manner is something that I now use daily, realizing that one should always plan for the future while remaining flexible.

Using this mindset of continual learning, I began to pursue research in the field of interventional radiology, focusing on quality improvement and new technique development. By working with residents at my medical school, we worked to minimize intraoperative radiation by standardizing pre-operative imaging review. Additionally, our team has worked on the introduction of augmented reality headsets in the procedure room to increase ease of access to patient imaging data. I aim to continue my work with my colleagues in biomedical engineering to introduce new techniques and technology, widening our arsenal and improving patient outcomes.

Fueled by my passion for advancing the field’s clinical efficiency and feasibility, I made a trip to my ethnic homeland in India to try to make a change there. I quickly discovered that access to facilities, personnel, and instruments was severely limited in rural areas. To combat this, I worked with a local interventional radiologist to devise a make-shift procedure room “on wheels”, allowing medical facilities and interventional radiologists to travel to places where they were most needed. Using an on-call schedule and working closely with other doctors in the area, we began to help patients that primary care identified as candidates for minor interventions! Our mobile unit also inspired other specialty services such as OBGYN and general surgeons to develop their mobile units as well!

As I near graduation, I realize that interventional radiology is where I can best apply my passion for problem-solving and leadership. Seeing all the moving parts, anticipating various outcomes and their probabilities, devising new strategies, and placing individuals where they work the best are just some of the things that are common between a chess player and an interventional radiologist. I look forward to realizing my goal to become a valuable asset to every patient I encounter and to inspire other physicians to push the boundaries of minimally invasive interventions, whether that is as a vital piece on the chess board in the hospital system, or the grandmaster chess player leading in the procedure room.

Residency Personal Statement Example #14:

Internal medicine/ icu | the farmer.

Growing up, my father’s dream for my future was that I would someday take over from him in running the family farm. My childhood was a continuous balancing act between completing homework, executing my farm duties, and being a good son to my parents. Years of navigating these competing responsibilities had made me fairly adept at multi-tasking, and in the back of my mind, I still harbored the fantasy that I could both fulfill my obligations to the farm while also entertaining my growing passion for medicine. However, this naïve, but well-intentioned vision for my future came crashing down when I was admitted to the hospital for meningitis. Spending days on end in the largest hospital in our city, I witnessed firsthand the impact of exceptional and compassionate patient care. I was impressed by the vast scientific knowledge and skillful manner in which my physician communicated my diagnosis and treatment plan with me. I knew then that I could never work on the family farm and that my true life’s calling was to become a physician.

For the next two years, I worked as a waiter to be able to afford my dream of attending medical school. Every day after a long shift at work, I would return home and study for the admissions exam until I fell asleep. After a grueling two years, I gained admission to medical school, thrilled to finally be studying the subject to which I had chosen to dedicate my life.

I quickly developed a passion for internal medicine as I began my clinical rotations, and in particular, the high-acuity patients I encountered in the intensive care unit. I was amazed by the medical complexity of each patient and the breadth of knowledge that critical care physicians must have in order to rapidly diagnose and treat patients, many of whom were hanging on to life by a thread. What I most enjoyed about my time rotating in the ICU was that almost every single patient was a medical puzzle, and that it took the concerted and deep collaboration of a whole team of healthcare providers to come to a suitable consensus on patient management. It was particularly awe-inspiring to see patients on the brink of death fully recover after spending a few days in the ICU. I quickly realized that I had found my intellectual and spiritual home, and that I would like nothing more than to dedicate my life to the care of the sickest patients in the hospital.

When I expressed my interest in pursuing internal medicine residency followed by a critical care fellowship to my mentor, she immediately recommended pursuing my dream through training in the US given the comparatively better access to cutting-edge technology, clinical experts, and seemingly limitless research opportunities. However, the financial burden was a huge barrier for me. I tackled this obstacle in the only way I had ever known how; by working in the evenings after school and on days off to save up enough money to come to the US. But even that was not enough to reach my goals, so I took on a job as a general practitioner in India for two years to be able to afford the plane tickets and the battery of exams needed for entry into US residency programs. This experience helped to hone my clinical skills and bedside manner and will serve me well during my residency training. Additionally, since coming to the US, I have become more involved in clinical research, working alongside critical care physicians at the Mayo Clinic on a number of projects and learning more about the intricacies of the US healthcare system.

Having spent two years in the US, I am ready to embark on the next step in my academic journey and look for a program with comprehensive internal medicine training and robust research infrastructure to expand my growing passion for clinical research. I aspire to be a clinician-scientist who takes insights from my interactions with patients in the ICU to further the field, both from a treatment perspective and from the perspective of improving health care equity and access.

My journey has been arduous, circuitous, and marked by many obstacles along the way. But I know of no other pathway as intellectually stimulating or personally rewarding as medicine. My father has since come to terms with his initial disappointment that I would not be taking up his mantle to work on the family farm. But he has expressed newfound pride in my goal to pursue medicine and to provide excellent care for patients and their families the way that the doctors that treated my meningitis did for me all those years ago. 

family medicine personal statement samples

Physician Advisor

A physician and native English speaker with exceptional expertise in revising residency personal statements.

family medicine personal statement samples

Structure Changes

By establishing smooth transitions, rearranging paragraphs, and advising on removing redundant details, we enhance the storytelling flow of your personal statement.

family medicine personal statement samples

Language Revision

By refining grammar, syntax, and word choice, we elevate the quality of your writing.

family medicine personal statement samples

Content Guidance

We provide significant detailed comments to ensure no important details are overlooked.

Residency Personal Statement Example #15:

General surgery | the iraqi female applicant.

‘Females can never be surgeons!’ These were the words that resonated in my ears every time I expressed my interest in surgery. My medical school tutors, family, friends, all dissuaded me from pursuing this course. In a patriarchal society like the one I grew up in, women were expected to adhere to restrictive cultural norms. Thankfully, I persevered.

Growing up in war-torn Iraq made for a difficult and unusual childhood. War and fighting were the norm, as were constant displacement and unstable living situations. Due to the unrelenting violence that ravaged the country since before I can remember, the emergency room in my medical school hospital, Al Mosul University Hospital, was constantly flooded with trauma patients.

The combination of diverse cases and shortage of clinical staff proved the perfect storm for piquing my surgical interests, as I was afforded the opportunity to perform tasks typically reserved for first and second-year residents. Though I quickly rose to the intense demands of working in Al Mosul’s ED, my male colleagues would often remind me that surgery was not an appropriate avenue for women, and that I should instead choose an ‘easier’ specialty that would allow me to focus on raising a family. For me, however, the decision was crystal clear. Surgery was the perfect blend of manual dexterity and methodical decision making. I was not only fascinated by the diversity of surgical cases, but also by the surgeons’ abilities to repair and heal the horrific war injuries. Seeing patients who suffered bomb blasts on the brink of death be stabilized through expert surgical intervention sparked my passion for the incredible restorative power of surgery. The fast pace, required precision, and the exquisite coordination of working as part of a surgical team further cemented my interest.

At a local surgical conference, I was fortunate to meet a visiting US surgeon who was in Mosul as part of his mission trip to Iraq. After speaking to him at length about my burgeoning interest in the field, he encouraged me to follow my passion, and even helped me secure several rotations in the US. It was during these rotations that I received my first exposure to the US healthcare system, from its incredible access to technological advancements unheard of in most Iraqi hospitals to its focus on cultivating a diverse and inclusive workforce. Following my rotations, I spent two years as a post-doctoral clinical researcher at Brigham and Women’s Hospital (BWH), investigating longitudinal outcomes for trauma patients who sustained debilitating war injuries. My research years were transformational, not only providing me a robust foundation in clinical research, but also giving me a deeper appreciation for the positive impact of holistic care on trauma patients’ lives and wellbeing. As a result of my experiences at BWH, I hope to enroll in a program with equal parts emphasis on surgical and research skills development and that embraces diversity as a core value. Following my residency, I aspire to return to Iraq and continue to treat patients suffering from trauma, conduct research on optimizing outcomes for trauma patients, and educating the next generation of surgeons.

As a female growing up in Iraq, I faced many challenges during my quest to secure a residency spot in the US. Despite the discouragement of tutors and family members as well as the daunting prospect of starting a long and difficult journey in a new country, I am steadfast in the pursuit of my professional dreams. I have one goal that I will keep fighting for in the years ahead: an unwavering commitment to make a difference in patients’ lives and empower women in Iraq and around the world to help me make that difference. My message to those women who, like me, are told by those around them that they can never be surgeons: do not be discouraged. Let their words fuel your strength and fight to make the world a better place for yourself and your patients!

Residency Personal Statement Example #16:

Pediatrics | the oncologist.

Walking into the pediatric ward for the first time was bittersweet. While it was sad to see that children so young have to be hospitalized and spend time away from their family and friends, I could also sense the unity with which the doctors, nurses, and other staff tried to make each child as happy as possible. Outpatient clinical encounters were the same: pediatricians would go out of their way to involve children in their healthcare without overwhelming them. Seeing the same patient with meningitis go from unresponsive one evening to a talkative and vibrant child in a few weeks was amazing. Unsurprisingly, when I started nearing the end of medical school, I was drawn towards pediatrics.

Just as the pediatricians tried to grant agency to scared and confused children, I started to make sure that I was doing the best I could to provide a sense of normalcy to the children I met during my pediatric rotation in my local hospital in Pakistan. Near the end of the rotation, I worked together with the nursing staff to provide ‘responsible cheat meals’ for kids who were sick of hospital food, repurposed childhood toys from my classmates, and provided earplugs to diminish noise from healthcare monitors during sleeping hours. In our limited and informal experience in the pediatric oncology ward, my preceptors and I found that this often made the children more cooperative and happier with their care, ultimately lowering the need for supportive medications such as analgesics.

However, one thing that I learned from this experience was that children are not always happy and receptive toddlers. Older teenagers in particular harbor a lot of skepticism towards healthcare professionals and need an extra level of attention. Working with an adolescent specialist at the University of Minnesota, I began to develop skills to help children deal with changes in their bodies and social expectations. This naturally led me to pursue the development of education programs for healthcare workers, parents, and teenagers on what to expect during puberty and beyond. We developed specific educational material for healthy children, special considerations during times of chronic illnesses such as cancer, as well as psychosocial techniques for communication.

My experiences in education also sparked an equal interest in research, as I noticed that there are many gaps in the literature regarding general predictors of mental well-being in the teenage oncological patient population. This led me to pursue a post-doctoral research position with Dr. John Smith at Boston Children’s Hospital, which focused on investigating the combined effect of chemoradiation therapy, baseline physical health, and social determinants on hospitalization rates in all forms of leukemia. Currently, we are working on using this data to develop cancer-specific risk assessment tools for mortality and long-term hospitalization to be integrated into daily clinical practice. While my medical school in Pakistan has afforded me with an amazing clinical education and exposure to a wide variety of pathology, my experience in the United States has motivated me to gain skills in education and research while also learning specialty-specific skills in pediatrics. A residency in the U.S. would allow me to hone these skills to serve a wider patient population.

As a pediatrics applicant, I aim to join a program that shares the same aims as I do: getting patients and parents out of the hospital as soon and as happy as possible! Long-term, I am particularly interested in pursuing a fellowship in pediatric oncology, fueled by my experiences in clinical practice, education, and research. By meeting individuals at their level, whether they are toddlers, teens, or parents, I hope to make a trip to the hospital an experience filled with optimism.

Need guidance on crafting that perfect personal statement? Swing by our blog “ How to Write a Good Personal Statement for your Residency Application ” for a fun walkthrough on creating a standout residency application statement.

family medicine personal statement samples

Residency Personal Statement Example #17:

Emergency medicine | the firefighter.

For as long as I can recall, it seemed my destiny was always to become a firefighter. Growing up as the son and grandson of two generations of City of Toledo Firefighters, I witnessed firsthand the selflessness and bravery displayed by these everyday heroes. They were the first responders who fearlessly confronted emergencies, rushing into flaming buildings and establishing deep connections with the community. It was their dedication that inspired me to follow in their footsteps. However, my path took an unexpected turn after high school when I decided to take a position working as an Emergency Medical Technician (EMT) prior to college.

During that transformative year, as I immersed myself in the world of emergency medical services, I had the privilege of interacting with emergency physicians both in the field and in the trauma bay. During these experiences, I was immediately captivated by their ability to think critically, remain calm in the face of chaos, and save lives. It was in those moments that I realized my true calling lay in the field of emergency medicine.

Coming from a blue-collar family, I understood the importance of hard work and determination. As the first person in my family to pursue a college degree, I enrolled in Owens Community College to pursue an Associate’s Degree in Pre-medicine. During this time, I continued to work as an EMT on weekends and during summers, financing my education through steadfast commitment and sheer determination. After two demanding years at the community college, my efforts were rewarded when I earned a full scholarship to the University of Toledo to complete my bachelor’s degree before gaining admission to the Toledo School of Medicine.

From the moment I stepped into medical school, my decision to pursue emergency medicine remained resolute. However, I recognized the value of acquiring a comprehensive understanding of various medical disciplines, as emergency medicine demands proficiency in almost every aspect of medicine. I approached every clinical rotation with enthusiasm, eager to develop the diverse skill set required to excel in the dynamic environment of the emergency department.

As a testament to my passion for the field, I took the initiative to establish the University of Toledo’s Emergency Medicine Interest Group, creating a platform where like-minded individuals could come together. Through this group, I organized lunch talks by members of the department and facilitated shadowing opportunities for first and second-year medical students. Furthermore, I dedicated two months of elective time to work alongside emergency medicine residents and physicians during prehospital care rotations across Toledo, solidifying my passion for the specialty.

Looking ahead, I envision a future where I split my practice between a large teaching academic center and an underserved, rural community. In the academic center, I aim to contribute to the education of residents and students, sharing my experiences and expertise to shape the next generation of emergency physicians. Simultaneously, I am deeply committed to serving in a rural or underserved setting, where I can make a meaningful impact on the lives of those in need. I believe that everyone, regardless of their circumstances, deserves access to high-quality emergency care, and I am eager to provide comprehensive and compassionate medical services to underserved populations. With the unwavering motivation and dedication inherited from two generations of first responders, I am ready to embark on the next phase of my training in emergency medicine. 

Looking for a comprehensive ERAS Application Template with examples of diverse experiences and publication types?

Grab your FREE ERAS Template right here !

family medicine personal statement samples

Residency Personal Statement Example #18:

Internal medicine | the war survivor.

The Afghan Civil War erupted when I was in elementary school. Soon after, the Taliban occupied Afghanistan, and, as a girl, I was barred from my school. I had always dreamed of becoming the first female doctor in my family, and this was a goal that required extensive education, let alone elementary school. My family uprooted everything to migrate to Pakistan so that I would be able to continue my education. Living in a country where we were not welcomed, bearing financial burdens, and worrying about safety issues, especially for girls, were the least of the challenges we faced, but that did not hold me back.

Still, that was not the last challenge I faced. When I graduated high school, I could not afford to attend medical school in Pakistan. Instead, I accepted the offer to serve as a teacher at our community school. Teaching at such a young age, tutoring those similar in age to me, and managing a class of thirty students taught me a great deal of discipline and leadership, skills which I have since carried with me throughout my career.

A decade later, the Taliban regime was finally over. We returned to Afghanistan, and I attended the entrance exam for Kabul Medical University. Among thousands of other participants, I was part of the lucky 25% who passed the exam. My endurance had paid off. Finally in medical school, I found myself fascinated by the detailed knowledge and interdisciplinary approach of my internist attendings. Their synchronized orchestration of patient care resonated with my experiences managing diverse students, while their instructive whiteboard sessions on pathophysiology echoed my own tenure at the front of a classroom. These encounters served as enlightening examples, aiding me in sculpting my identity as a burgeoning physician.

On my internal medicine rotation, I was responsible for the care of a patient with multiple myeloma. His low hemoglobin level led to significant limitations in his daily activity. His symptoms were initially attributed solely to his condition, but I was not satisfied with this explanation. When I ordered his iron studies, we were able to diagnose him with concomitant iron deficiency anemia. An iron infusion quickly helped improve his quality of life, which was precious to my patient, as I knew from the time I had spent with him. That ability to help my patient made me finally feel like the doctor I aspired to be. I had found my home in internal medicine. The convergence of laboratory tests, imaging studies, and critical analysis to reach a diagnosis fuels my desire to become an internist.

Despite my passion for internal medicine, women in Afghanistan faced scant opportunities in this field. This was due to a lack of female mentors and sociocultural constraints against females being on night shifts in predominantly male hospitals. Undeterred, I embarked on another journey away from home, this time to the United States. Here, I secured a position as a medical scribe, working in tandem with various healthcare providers. This experience allowed me to absorb their expertise, familiarize myself with the U.S. healthcare system, and diligently prepare for and ultimately pass the USMLE exams.

I have come a long way, and still have a long way to go. My accomplishment of becoming my family’s first female doctor fills me with pride. Yet, I aspire to achieve more – to become a distinguished internist and an empowering role model for the women of Afghanistan. I intend to personify the belief: if you dare to dream, you are destined to achieve. 

Residency Personal Statement Example #19:

Internal medicine | changing specialties.

When I was a senior in high school, my girl scout troop would organize weekly medical trips to rural parts of our community, working with local physicians to provide basic medical services to underserved patient populations. I was particularly struck by the excellent care and bedside manner of one of the physicians who used his bilingualism to connect with a non-English speaking patient who had faced significant challenges in accessing care. The doctor’s small gesture left a lasting impression on me, and, for the first time, I realized not only the curative but also the humanistic power of medicine to connect with patients across cultural barriers and in some of their most vulnerable moments. Though I had always had a proclivity for science, it was not until that moment that I had ever seriously considered a career in medicine.

In medical school, I was captivated by pre-clinical coursework in pathology and lectures on disease pathophysiology. I was torn between pathology and internal medicine during my clinical rotations, as I enjoyed the cerebral, deductive nature of each field and the fact that neither was limited to a single organ system or patient population. The opportunity to be the frontline diagnostician and to utilize advanced equipment and laboratory methods eventually won me over to pathology.

However, during my pathology residency, the pendulum started to swing back toward internal medicine. I vividly remember the turning point in my decision making. I was staring down the barrel of my microscope at dozens of Plasmodium falciparum gametocytes on a peripheral blood smear. I paged the internal medicine team to help confirm the diagnosis of cerebral malaria. Hearing the excitement and celebration of the medical team on the other end, who had been struggling to identify the etiology of the patient’s undulant fevers and fatigue, I felt a pang of envy, a distinct feeling that I was missing out on the human factor of medicine.

Similarly, in my research on the utility of galectin-3 immunohistochemistry staining in papillary carcinoma of the thyroid, I found myself increasingly drawn to the human impacts of scientific investigation. For example, after my successful completion of several experiments, our department was able to secure funding to examine a wider range of malignancies. I was particularly excited when my research enabled our hospital to offer estrogen and progesterone receptor testing and hormonal therapy for breast cancer patients. I quickly realized that I did not just want to diagnose but to directly treat patients, and with each passing day, I yearned more for the ability to heal through empathic listening and the formation of meaningful rapport with patients.

Eventually, I decided to undertake the goal of retraining in internal medicine. To this end, I elected to travel to the United States to undertake hands-on clinical experiences. My time in the U.S. gave me firsthand exposure to a complex healthcare system and a deeper appreciation for the impact of advanced diagnostic technology, cutting-edge treatment modalities, and patient-centered, evidence-based care. I also gained confidence in my abilities to function as a member of a large, interdisciplinary care team, drawing on a skillset I had cultivated from many years of leading my girl scout troop and performing in church choirs.

I aspire to enter a residency program with an emphasis on strong clinical skills training, excellent research opportunities, and a dedication to clinical mentorship. Moreover, I want to be part of a program with strong camaraderie among residents and faculty and a spirit of collegiality and tireless dedication to patient care. Ultimately, I believe that my background in and extensive knowledge of pathology, my compassionate disposition, and my penchant for diligence and collaboration will make me a strong applicant to your residency program. Thank you for your consideration of my application. 

Residency Personal Statement Example #20:

General surgery | the colombian img.

From the coastlines of Colombia, where I grew up assisting my mother—a nurse at our local clinic—during community emergencies, to the ORs of the United States, my journey has been driven by a single purpose: to master the art of surgery. My childhood in a region frequently struck by natural disasters exposed me to the critical need for deliberate, effective medical interventions. These early experiences ignited my passion for surgery, the field where I believed I could make the most immediate impact.

I pursued medical training in Bogotá, completing medical school and a residency in general surgery, where I became adept at navigating the complexities of trauma care under resource constraints. This foundational experience instilled in me a deep understanding of the vital role of precision and innovation in saving lives, yet it also highlighted the limitations imposed by a lack of advanced technology.

Determined to push the boundaries of what I could offer my patients and at the urging of my clinical mentors, I sought advanced training in the United States. Passing the USMLE was a challenging yet rewarding milestone. Next, after sending 100s of emails I eventually obtained a research fellowship at Jefferson University Hospital in Philadelphia. Under the mentorship of Dr. Elizabeth Hansen, a leader in robotic surgery, I delved into the intricacies of robotic-assisted surgical techniques, contributing to research that sought to enhance surgical precision and safety. This work not only expanded my technical expertise but also fueled my passion for innovation, culminating in multiple publications and presentations at national conferences. These experiences solidified my commitment to surgical excellence and my desire to lead advancements in the field.

My clinical rotation at Cleveland Clinic under Dr. Michael Choi, a pioneer in minimally invasive surgery, was particularly formative. Here, I honed my skills in laparoscopic procedures and participated in a study focusing on the application of these techniques in emergency surgeries. Our work demonstrated significant reductions in patient recovery times and was recently published in the Journal of Trauma and Acute Care Surgery.

Looking to the future, I am driven by a vision to transform surgical care in underserved regions, starting with my home country, Colombia. In the short term, I hope to match into a strong general surgery program to continue to hone my clinical skills. Though I remain open, I am inclined to pursue further fellowship training in minimally invasive and robotic surgery. My end goal is to establish a center of excellence for minimally invasive surgery, where I can train a new generation of surgeons in advanced techniques that are adaptable to both high-tech environments and resource-limited settings.

The United States has offered me unparalleled opportunities to grow as a surgeon and a scholar. However, the essence of my journey remains rooted in my early experiences in Colombia—facing adversity with limited resources but abundant resolve. I am eager to join a residency program that values not only technical skills but also the drive to apply those skills in diverse and challenging environments. I am committed to becoming not just a surgeon, but a global surgical leader, enhancing the quality and accessibility of surgical care worldwide. 

family medicine personal statement samples

Advisor UNLIMITED Access

We get how stressful the residency match process is, so we're here for you - communicate with your personal advisor ANYTIME you need!

Residency Personal Statement Example #21:

Emergency medicine | the flow.

Anybody who has ever played at a jam session can tell you that we all live for the flow state: that state of mind during which you can place every improvised note well before you play it, and where you can perfectly see where you fit in with every other member of your band. I found that working in the emergency room on a busy day, I could feel the same flow-state as running codes and triaging patients, deciding how to deal with whatever comes through those doors optimally. This marked the start of my journey to becoming an emergency physician.

Nothing cemented my decision to pursue this field more than when an earthquake devastated my hometown in Sri Lanka, resulting in an overcrowded emergency department for more than a week as we appropriately managed anyone coming through the door. Daily, we had pre-rounds with local authorities about expected numbers and resource management. Next, we divided the list into emergent, urgent, and stable patients and began tackling all tasks ranging from splinting simple fractures to complex multi-compartment trauma. Finally, this all occurred over our regular influx of individuals with heart attacks, drug overdoses, and other acute presentations. While it was a truly grueling experience, I discovered that once I got into the rhythm of things, managing patients became easier and easier and I found myself eagerly asking ‘What needs to be done next?’

As exhilarating as this experience was, I understood from my experience that we were thankfully adequately staffed for the situation with an appropriate number of supplies. From my discussions with healthcare professionals from other institutions, this is not always the case. To combat this issue, we assembled the leadership of several local hospitals to define what it means by a ‘local emergency’, and devise resource-sharing hotlines, and post-emergency debriefings. With this system, we hope to timely redirect patients to hospitals with appropriate resources in the event of future catastrophes. Indeed, we found that this system eventually helped us with a completely different sort of emergency in the COVID pandemic where cross-institutional training helped us tide the initial waves.

My conversations with other emergency personnel also revealed another aspect of emergency medicine that I felt I had not experienced: being a first responder. To understand the perspective of the healthcare professionals who are first on the scene, I joined a paramedic team that responded to stroke calls, heart attacks, trauma, and other such emergencies. Here, communication between the destination hospital and initial patient management needs to be juggled in a time-effective manner. With this experience, I now better realize what emergency departments can do to make first responders’ jobs easier, which can be as complex as coordinating multi-service consults to as simple as skipping the ER directly to take the patient to the catheterization lab.

As a musician, I understand that working in a team cannot be a one-man show with guitar solos all the time. The same principle applies in the ER, where sometimes you are the person best equipped for a certain situation but need to take a backseat to other experts in other scenarios. Regardless of my role, I aim to be an asset to any team of emergency healthcare professionals by honing my skills, responding to team dynamics collegially, and yearning to make the lives of first responders everywhere easier.

If you are applying to the Match and need a detailed guide on how to ace your ERAS application, check out our free ERAS Application Guide here . 

Residency Personal Statement Example #22:

Primary care/im | the impoverished.

“If you can dream it, you can achieve it.” These powerful words, spoken by my mother, have echoed in my mind since childhood. Growing up in a single-parent home on the south side of Chicago, my mother worked tirelessly as a nurse in Advocate Good Samaritan Hospital’s emergency department. Every night my brother and I would wait for her to arrive after her shift ended at 7 pm. As she shared stories of dedicated physicians and life-saving interventions, I began to view these doctors in the same manner my friends viewed superheroes or sports stars, inspiring me to pursue a career in medicine.

As an African American in a neighborhood lacking professional role models, the path to becoming a physician seemed distant if not impossible. However, my mother’s belief in the power of dreams instilled in me the courage to strive for the extraordinary. With determination, I worked diligently throughout grade school and middle school, propelled by the aspiration to transcend the limitations of my circumstances. Eventually, I was admitted to Whitney M. Young Magnet High School, a magnet school named after a civil rights activist and one of my personal heroes.

Continuing to embrace every opportunity, I was able to attend Northwestern University on a full academic scholarship, where I immersed myself in neuroscience studies. Additionally, I dedicated my time as a tutor, providing support to underserved children in my former neighborhood. Witnessing the impact of education and healthcare disparities further ignited my passion for addressing these inequities.

Entering the University of Chicago Medical School, I embarked on a transformative journey. During my third-year clerkships, I discovered my calling in primary care and internal medicine. Although initially drawn to the fast-paced environment of the emergency department, I found the thoughtful, cerebral approach of internal medicine captivating. Each day, I eagerly embraced the challenge of unraveling complex medical puzzles, weaving together a patient’s diverse comorbidities to form a comprehensive list of differential diagnoses.

Following my third year, I took a gap year dedicated to serving underserved populations in Chicago. This experience provided a profound understanding of social determinants of health and the importance of preventive medicine. It solidified my commitment to bridging the gaps in healthcare access and outcomes, particularly within urban communities like my own. Looking forward, my vision encompasses practicing as a primary care physician in an urban academic center, where I can not only provide compassionate patient care but also mentor and inspire medical students and residents. Furthermore, I aspire to conduct research that addresses social determinants of health, striving to make tangible improvements in my community.

Reflecting on my journey, I realize that my mother’s quote encapsulates the essence of my pursuit. With each step I’ve taken, from the dinner table conversations with my mother to my experiences in medical school, I have seen firsthand that dreams can indeed be transformed into reality. By embracing the challenges, dedicating myself to lifelong learning, and advocating for equitable healthcare, I am ready to embark on a fulfilling career in internal medicine—a path that resonates with my values, aspirations, and the indomitable spirit instilled in me by my remarkable mother. “If you can dream it, you can achieve it.” These words, once whispered to me at the beginning of my journey, now reverberate with even greater significance as I stand at the threshold of a future where I can make a lasting difference in the lives of others. 

Looking for a detailed ERAS Application Template with samples of various experiences?

Claim your FREE ERAS Template here !

Residency Personal Statement Example #23:

Internal medicine | nonna.

“Not too much parmigiano, angioletta,” Nonna would call from the head of the table as I layered cutlets, marinara, and cheese. At ten, I became her hands in the kitchen, after diabetic peripheral neuropathy had stolen the fine motor strength and sensation needed to construct a perfect chicken parmesan. In Nonna’s kitchen, somewhere between deglazing the pan and helping prepare her insulin injections, I discovered a passion even more fervent than my love of home-cooked Italian food. The transformative power of medicine captivated me, and I became dedicated to pursuing a career that could allow me to help other patients with lifelong diseases like diabetes.

Throughout my clinical education, all my patient encounters reinforced the impact of compassionate, comprehensive healthcare. On my internal medicine rotation, I fell in love with the diagnostic aspect of the field, as well. For example, I had a 34-year-old patient with a seemingly unprovoked pulmonary embolism and incidental hydronephrosis on CT. After digging a little bit, I found out he had a history of cryptorchidism with unilateral orchiectomy, and I wasn’t convinced he was clotting without provocation. Testicular ultrasound showed a mass that had entirely replaced the normal testicular tissue, and CT of the abdomen and pelvis showed a lymph node compressing the ureter. I realized that this was how I wanted to spend the rest of my career—proposing a unifying diagnosis through careful interpretation of data to find patient-centered interventions.

Even the more routine aspects of medicine were exciting to me. I woke up before my alarm each morning, excited to interpret new lab data from the night before and preround on my patients to see if they were improving or needed further care. I was especially excited to participate in patient education. I spent two hours counseling one of my patients with diabetes and a new insulin requirement on the logistics and barriers of self-injection. While she was admitted with incredible resistance to the idea of insulin injections, I spent each day of admission trying to understand and resolve her fears. By discharge, she was able to self-administer basal and bolus. I have witnessed firsthand the importance of empathy in establishing trust and fostering meaningful patient-provider relationships, qualities I strive to embody in my practice.

As a South Philadelphia native, I see the social determinants of health at work in my backyard every day. I am passionate about addressing how these factors impact the delivery of care. I am deeply committed to advocating for health equity and addressing the social determinants of health that disproportionately affect marginalized communities. Specifically, I spent all four years of medical school volunteering at a student-run clinic, providing free medical care to unhoused people in Philadelphia. At this clinic, I developed an education program on commonly seen conditions which is now held annually for the residents of the shelter. In clinical practice and beyond, I am dedicated to improving health literacy and access to care for all my patients.

My academic pursuits have complemented these experiences, providing me with a solid foundation in evidence-based medicine and critical thinking. For example, I have completed a research project on the barriers to insulin distribution and injection, and the potential role of social media as an educational intervention for younger adults who are newly diagnosed. I am invested in advancing the field of internal medicine through clinical research and hope to spend my residency and career continuing to contribute to the field.

In a residency program, I am seeking to join a team with the same values I learned from Nonna almost fifteen years ago: community, supportive learning, and awe of the work we do. I hope to match at an institution that will prioritize fostering an awareness of social determinants of health and emphasize patient-centered care above all else. I am eager to continue my professional growth under the guidance of experienced clinicians and educators who share my empathy-forward approach and commitment to advancing the field for the benefit of our patients. 

Final Thoughts

Hopefully, these samples will help you draft an excellent personal statement to tell the great story of your medical journey!

If you need help with editing your personal statement or having an expert lay an eye on it and give you comprehensive feedback, don’t hesitate to reach out to us  HERE !

You can also bundle your personal statement editing with ERAS application editing and interview preparation by signing up to our Match Application Packages HERE .

If you have any questions about any of our services, don’t hesitate to reach out to our customer support service here .

Good luck with your application and always remember, The Match Guy is here for you!

To your Match, The Match Guy

family medicine personal statement samples

2025 ERAS Application Updates 

General Surgery Residency Personal Statement Examples

General Surgery Residency Personal Statement Examples 

Emergency Medicine Residency Personal Statement Examples

Emergency Medicine Residency Personal Statement Examples 

Internal Medicine Residency Personal Statement Examples

Internal Medicine Residency Personal Statement Examples 

About thematchguy, recommended articles.

family medicine personal statement samples

ERAS Application 2024 Guide!

family medicine personal statement samples

How To Write A Good Personal Statement For Your Residency

family medicine personal statement samples

Medical Student Performance Evaluation for Residency Applicants 

family medicine personal statement samples

Letters of Interest for Residency Applicants

How can we help you.

Leave your message here and we will get in touch with you as soon as possible.

family medicine personal statement samples

Quick Links

  • Our Reviews
  • Our Podcast
  • Personal Statement
  • Match®Application Package
  • P.O. Box 40388 Pittsburgh, PA 15201
  • [email protected]
  • + 1 412-295-8358
  • Privacy Policy
  • Terms & Conditions

WhatsApp Us

family medicine personal statement samples

  • (888) 381-9509
  • [email protected]
  • Book a Meeting
  • student login
  • Student Login
  • Our Services
  • Our Story How it started
  • Our Team Meet Our Advisors & Tutors
  • Our Services How we can help you
  • Our Difference Learn why we stand out
  • Success Stories & Testimonials Hear the stories
  • For Parents Learn why you should trust us
  • In the News Read Our Stories
  • Frequently Asked Questions Find answers
  • MCAT Tutoring One-on-One Personalized Help
  • MCAT Go An Audio Learning Experience
  • MCAT Practice Exams Boost Your Score
  • MCAT Prep App Videos, Flashcards & Q-Bank
  • MCAT CARS Mastery Top-Rated CARS Video Course
  • Pre-Med Coach Early High School Roadmap Planning
  • College Admissions 11th & 12th Grade Pre-Med Consulting
  • Direct Med Advising BS/MD Application Support
  • Interview Preparation BS/MD Candidates
  • Pre-Med Coach Pre-Application Development
  • Application Advising Med School Admissions Support
  • Personal Statement Editing Refine Your Narrative
  • AMCAS Editing Application Editing
  • Secondary Editing Secondary Application Editing
  • Interview Preparation Realistic Practice
  • CASPer Preparation Simulation & Coaching
  • Ontario Application Support OMSAS Application
  • Residency Advising Complete Match Support
  • Residency Interview Preparation
  • ERAS Personal Statement Refine Your Story
  • USMLE STEP 1 Maximize your scores
  • USMLE STEP 2 Shine on your boards
  • USMLE STEP 3 Conquer your final hurdle
  • COMLEX LEVEL 1 and 2 Score higher
  • Institutional Partners Enhance your student offering
  • Organizational Partners Provide value to your students
  • Virtual Shadowing Explore Medical Specialties
  • Extracurricular Activities Apply now!
  • Under the Stethoscope Admissions Video Course
  • Research Roadmap Master Clinical Research
  • MSC Score Calculate Your Chances
  • Guidebooks Comprehensive Guides
  • Med School Explorer Find Your School
  • MCAT Masterclass Videos, Questions, Notes
  • Anatomy Anki Deck Practice with 4,000+ Flaschards

The Only 3 Medical School Personal Statement Examples You Need to Read

family medicine personal statement samples

Posted in: Applying to Medical School

family medicine personal statement samples

Table of Contents

The personal statement is one of the most important parts of the med school application process because t his mini-essay is a critical opportunity for you to stand out from other prospective medical students by demonstrating your passion and personality, not just your grades.

Admissions committees receive hundreds or more AMCAS medical school applications , so yours should be unique and captivating. Your medical school personal statement shows admissions officers who you are beyond your high school or pre-med GPA , extracurriculars , and MCAT score . 

The best personal statements are… well, personal . This is your chance to share what life experiences have compelled you toward a career in healthcare or the medical field , and how those experiences shape the picture of your ideal future.

MedSchoolCoach has crucial advice for writing your personal statement . 

Read these examples of personal statements for prospective med students.

Writing a great medical school personal statement is a lot easier with the right support. We’ve helped numerous med school applicants craft top-notch personal statements and can do the same for you.

But first: 7 steps to writing an engaging personal statement.

Before you read these excellent examples, you need to understand the process of writing a personal statement.  

Include these in your medical school personal statement:

  • Why you’re passionate about becoming a doctor
  • Your qualities that will make you a great physician
  • Personal stories that demonstrate those qualities
  • Specific examples of the communities you want to serve as a member of the medical field

What are the most important things to remember when writing a medical school personal statement ?

  • Begin the writing process early: Give yourself plenty of time for brainstorming and to revisit your first draft, revising it based on input from family members and undergrad professors. Consult the application timeline for your target enrollment season.
  • Choose a central theme: An unfocused essay will leave readers confused and uninterested. Give your statement a clear thesis in the first paragraph that guides its formation.
  • Start with a hook: Grab the reader’s attention immediately with your statement’s first sentence. Instead of opening with a conventional introduction, be creative! Begin with something unexpected.
  • Be the you of today, not the you of the future: Forecasting your future as a physician can come across as empty promises. Don’t get caught up in your ambitions; instead, be honest about your current situation and interest in the field of medicine.
  • Demonstrate your passion: It’s not enough to simply state your interest in becoming a doctor; you have to prove it through personal stories. Show how your perspectives have been shaped by formative experiences and how those will make you an effective physician.
  • Show, don’t tell : Avoid cliches that admissions committees have heard hundreds of times, like “I want to help people.” Make your writing come alive with dynamic, persuasive storytelling that recounts your personal experiences.
  • Tie everything together: Conclude by wrapping up your main points. Reiterate your passion for the medical profession, your defining personal qualities, and why you’ll make a good doctor.

You can read more about our recommended method in our step-by-step guide , but those are the major points.

Example 1 — From the Stretcher to the Spotlight: My Journey to Becoming an Emergency Medicine Physician

Another siren shrieks as the emergency room doors slide open and a team of EMTs pushes a blood-soaked stretcher through the entrance. It’s the fifth ambulance to arrive tonight — and only my first clinical shadowing experience in an emergency medicine department since my premed education began.

But it wasn’t my first time in an emergency room, and I knew I was meant to be here again.

In those crucial moments on the ER floor, many of my peers learned that they stumble in high-pressure environments. A few weeks of gunshot wounds, drug overdoses, broken bones, and deep lacerations in the busiest trauma bay in the region were enough to alter their career path.

They will be better practitioners somewhere predictable, like a pediatrician in a private practice where they choose their schedules, clients, and staff.

Every healthcare provider has their specialties, and mine are on full display in those crucial moments of lifesaving care. Why am I pursuing a career in Emergency Medicine? Because I’ve seen firsthand the miracles that Emergency Medicine physicians perform.

12 years ago, I was in an emergency room… but I was the one on the stretcher.

A forest-green Saturn coupe rolled into my parent’s driveway. The driver, my best friend Kevin, had just passed his driving test and was itching to take a late-night run to the other side of town. I had ridden with Kevin and his father many times before when he held his learner’s permit. But this time, we didn’t have an adult with us, and the joyride ended differently: with a 40-mph passenger-side collision, T-boned by a drunk driver.

I distinctly recall the sensation of being lifted out of the crumpled car by a paramedic and laid onto a stretcher. A quick drive later, I was in the care of Dr. Smith, the ER resident on call that night. Without missing a beat, he assessed my condition and provided the care I needed. When my mom thanked him for saving my life, he simply responded, “It’s what he needed.”

Now I’m watching other doctors and nurses provide this life-saving care as I observe as a premed student. I see the way the staff works together like a well-oiled machine, and it reminds me of my time in high-school theater.

Everyone has a role to play, however big or small, to make the show a success. All contributions are essential to a winning performance — even the technicians working behind the scenes. That’s what true teamwork is, and I see that same dynamic in the emergency department.

Some actors freeze during performances, overcome by stage fright. Other students are too anxious to even set foot in front of an audience; they remain backstage assisting with split-second costume changes.

Not me. I felt energized under the spotlight, deftly improvising to help my co-stars when they would forget their lines. Admittedly, I wasn’t the best actor or singer in the cast, but I provided something essential: assurance under pressure. Everyone knew me as dependable, always in their corner when something went awry. I had a reputation for remaining calm and thinking on my feet.

My ability to stay unruffled under pressure was first discovered on stage, but I can use it on a very different platform providing patient care. Now, when other people freeze under the intensity of serving public health on the front lines, I can step in and provide my calm, collected guidance to see them through.

As an ER doctor, I will have to provide that stability when a nurse gets flustered by a quarrelsome patient or shaken from an irreparably injured infant. When you’re an Emergency Medicine physician, you’re not following a script. It takes an aptitude of thinking on your toes to face the fast pace and unpredictable challenges of an emergency center.

During my time shadowing, I saw experienced physicians put those assured, gentle communication skills to use. A 13-year-old boy was admitted for a knife wound he’d received on the streets. He only spoke Spanish, but it was clear he mistrusted doctors and was alarmed by the situation. In mere minutes, one of the doctors calmed the patient so he could receive care he needed.

Let me be clear: I haven’t simply gravitated toward Emergency Medicine because I liked it most. It’s not the adrenaline or the pride that compel me. I owe Emergency Medicine my life, and I want to use my life to extend the lives of other people. Every person brought into the trauma bay could be another me , no matter what they look like.

People are more than their injury, health record, or circumstances. They are not just a task to complete or a challenge to conquer.

My childhood injury gave me an appreciation for the work of ER doctors and a compassion for patients, to foster well-being when people are most broken and vulnerable. I already have the dedication to the work and the heart for patients; I just need the medical knowledge and procedural skills to perform life-saving interventions. My ability to remain calm, think on my toes, be part of a team, and work decisively without making mistakes or overlooking critical issues will serve me well as an Emergency Medicine physician.

Some ER physicians I spoke with liked to think that they’re “a different breed” than other medical professionals — but I don’t see it that way. We’re just performing a different role than the rest of the cast.

Breaking It Down

Let’s look at what qualities make this a great personal statement for med school.

  • Engaging opening: The writer painted a vivid scene that immediately puts the reader in their shoes and leaves them wanting more.
  • Personal examples: The writer demonstrated his ability to stay calm, work as a team, and problem-solve through theater experience, which he also uses as a comparison. And, he explained his passion for Emergency Medical care from his childhood accident.
  • Organized: The writer transitions fluidly between body paragraphs, connecting stories and ideas by emphasizing parallels and hopping back and forth between time.
  • Ample length: Makes full use of the AACOMAS and AMCAS application personal statement’s character limit of 5,300 characters (including spaces), which is about 850-950 words.

Unsure what traits and clinical or research experience your preferred medical school values ? You can research their admissions requirements and mission statement using the MSAR .

Example 2 — Early Clinical Work For Empathetic Patient Care

The applicant who wrote this personal statement was accepted into University of South Florida Morsani College of Medicine, University of Central Florida College of Medicine, and Tufts University School of Medicine.

As I walked briskly down the hall to keep up during our daily rounds in the ICU, I heard the steady beeping of Michelle’s cardiac monitor and saw a ruby ornament twinkling on the small Christmas tree beside her. She was always alone, but someone had decorated her room for the holidays.

It warmed my heart that I wasn’t the only one who saw her as more than a patient in a coma. I continually felt guilty that I couldn’t spend more time with her; her usual companions were ventilators, IV bags, and catheters, not to mention the golf ball-sized tumors along her spine. Every day, I thought about running to Michelle’s bedside to do anything I could for her.

Thus, I was taken aback when my advisor, who was visiting me that day, asked me if I was okay. It never crossed my mind that at age 17, my peers might not be able to handle the tragedies that healthcare workers consistently face. These situations were difficult, but they invoked humanity and compassion from me. I knew I wanted to pursue medicine. And I knew I could do it.

From my senior year of high school to my senior year of college, I continued to explore my passion for patient interaction.

At the Stepp Lab, I was charged with contacting potential study participants for a study focusing on speech symptoms in individuals with Parkinson’s Disease. The study would help future patients, but I couldn’t help but think: “What are we doing for these patients in return?” I worried that the heart and soul behind the research would get lost in the mix of acoustic data and participant ID numbers.

But my fears were put to rest by Richard, the self-proclaimed “Parkinson’s Song & Dance Man,” who recorded himself singing show tunes as part of his therapy. Knowing that he was legally blind and unable to read caller ID, I was always thrilled when he recognized my voice. The spirit in his voice indicated that my interest in him and his journey with Parkinson’s was meaningful. Talking with him inspired me to dive deeper, which led to an appreciative understanding of his time as a sergeant in the U.S. military.

It was an important reminder: my interest and care are just as important as an effective prescribed treatment plan.

Following graduation, I began my work as a medical assistant for a dermatologist. My experience with a patient, Joann, validated my ability to provide excellent hands-on patient care. Other physicians prescribed her painkillers to relieve the excruciating pain from the shingles rash, which presented as a fiery trail of blisters wrapped around her torso. But these painkillers offered no relief and made her so drowsy that she fell one night on the way to the bathroom.

Joann was tired, suffering, and beaten down. The lidocaine patches we initially prescribed would be a much safer option, but I refused for her to pay $250, as she was on the brink of losing her job. When she returned to the office a week later, she held my hand and cried tears of joy because I found her affordable patches, which helped her pain without the systemic effects.

The joy that pierced through the weariness in her eyes immediately confirmed that direct patient care like this was what I was meant to do. As I passed her a tissue, I felt ecstatic that I could make such a difference, and I sought to do more.

Since graduation, I have been volunteering at Open Door, a small pantry that serves a primarily Hispanic community of lower socioeconomic families. It is gut-wrenching to explain that we cannot give them certain items when our stock is low. After all, the fresh fruits and vegetables I serve are fundamental to their culturally-inspired meals.

For the first time, I found myself serving anguish rather than a helping hand. Usually, uplifting moments strengthen one’s desire to become a physician, but in this case, it was my ability to handle the low points that reignited my passion for aiding others.

After running out of produce one day, I was confused as to why a woman thanked me. Through translation by a fellow volunteer, I learned it was because of my positivity. She taught me that the way I approach unfavorable situations affects another’s perception and that my spirited attitude breaks through language barriers.

This volunteer work served as a wake-up call to the unacceptable fact that U.S. citizens’ health suffers due to lack of access to healthy foods. If someone cannot afford healthy foods, they may not have access to healthcare. In the future, I want to partner with other food banks to offer free services like blood pressure readings. I have always wanted to help people, but I now have a particular interest in bringing help to people who cannot afford it.

While the foundation of medicine is scientific knowledge, the foundation of healthcare is the word “care” itself. I never found out what happened to Michelle and her Christmas tree, but I still wonder about her to this day, and she has strengthened my passion to serve others. A sense of excitement and comfort stems from knowing that I will be there for people on their worst days, since I have already seen the impact my support has had.

In my mind, becoming a physician is not a choice but a natural next step to continue bringing humanity and compassion to those around me.

How did this personal statement grab and sustain attention so well?

  • Personalization: Everything about this statement helps you to understand the writer, from their personal experiences to their hope for how their future career will look.
  • Showing, not telling: From the first sentence, the reader is hooked. This prospective medical student has plenty of great “on paper” experience (early shadowing, clinical experience, etc.), but they showed this with storytelling, not by repeating their CV.
  • Empathy: An admissions committee reading this personal statement would know beyond a shadow of a doubt that this student cares deeply about their patients. They remember first names, individual details, and the emotions that each patient made them feel.
  • A clear path forward: The writer doesn’t just want to work in the medical field — they have a passion for exactly how they want to impact the communities they serve. Outside of strictly medical work, they care about the way finances can limit access to healthcare and the struggle to find healthy food in food deserts around the US .

Read Next: How Hard Is It to Get Into Medical School?

Example 3 — Beyond the Diagnosis: The Importance of Individualized Care in Medicine

The applicant who wrote this personal statement was accepted into Touro College of Osteopathic Medicine and Nova Southeastern University College Of Osteopathic Medicine.

Dr. Haywood sighs and shakes her head upon opening the chart. “I was worried about her A1C. It’s up again. Hypertension, too. Alright, let’s go.”

As we enter the patient’s room, I’m expecting the news about her blood sugar and pressure to fill the room. Instead, Dr. Haywood says, “Roseline! How are you doing? How’s your girl, doing well?”

Dr. Haywood continues to ask questions, genuinely interested in Roseline’s experience as a new mother. If not for the parchment-lined examination chair and anatomy posters plastered to the wall, this exchange could be happening in a grocery store. What about her A1C? Her blood pressure? Potential Type II diabetes?

As I continue to listen, Dr. Haywood discovers that Roseline’s mother moved in with her, cooking Haitian meals I recognize as high on the glycemic index. Dr. Haywood effortlessly evolves their conversation to focus on these. Being Haitian herself, she knows some traditional dishes are healthier than others and advises Roseline to avoid those that might exacerbate her high blood sugar and blood pressure. Dr. Haywood also suggests Roseline incorporate exercise by bringing her baby on a walk through her neighborhood.

During my shadowing experience, I observed one of the core components of being a physician through several encounters like this one. By establishing a relationship with her patient where Roseline was comfortable sharing the details of new motherhood, Dr. Haywood was able to individualize her approach to lowering the patient’s A1C and hypertension. Inspired by her ability to treat the whole person , I began to adopt a similar practice as a tutor for elementary kids in underserved areas of D.C.

Shaniyah did not like Zoom, or math for that matter. When I first met her as a prospective tutee online, she preferred to keep her microphone muted and would claim she was finished with her math homework after barely attempting the first problem. Realizing that basing our sessions solely on math would be fruitless, I adapted my tutoring style to incorporate some of the things for which she had a natural affinity.

The first step was acknowledging the difficulties a virtual environment posed to effective communication, particularly the ease at which distractions might take over. After sharing this with Shaniyah, she immediately disclosed her struggles to share her work with me. With this information, I found an online platform that allowed us to visualize each other’s work.

This obstacle in communication overcome, Shaniyah felt more comfortable sharing details about herself that I utilized as her tutor. Her love of soccer gave me the idea to use the concept of goal scoring to help with addition, and soon Shaniyah’s math skills and enthusiasm began to improve. As our relationship grew, so did her successes, and I suspect the feelings I experienced as her tutor are the same as a physician’s when their patient responds well to prescribed treatment.

I believe this skill, caring for someone as a whole person , that I have learned and practiced through shadowing and tutoring is the central tenet of medicine that allows a doctor to successfully treat their patients.

Inspired by talking with patients who had received life-altering organ transplants during my shadowing experience, I created a club called D.C. Donors for Georgetown University students to encourage their peers to register as organ donors or donate blood. This experience taught me that to truly serve a person, you must involve your whole person, too.

In starting this club to help those in need of transplants, I had to dedicate my time and effort beyond just my physical interactions with these patients. For instance, this involved reaching out to D.C.’s organ procurement organization to inquire about a potential partnership with my club, to which they agreed. In addition, I organized tabling events on campus, which required significant planning and communication with both club members and my university.

Though exciting, starting a club was also a difficult process, especially given the limitations the pandemic imposed on in-person meetings and events. To adapt, I had to plan more engaging meetings, designing virtual activities to make members more comfortable contributing their ideas. In addition, planning a blood drive required extensive communication with my university to ensure the safety of the staff and participants during the pandemic.

Ultimately, I believe these behind-the-scenes actions were instrumental in addressing the need for organ and blood donors in the D.C. area.

From these experiences, I have grown to believe that good medicine not only necessitates the physician cares for her patient as a whole, but also that she fully commits her whole person to the care of the patient. Tutoring and starting D.C. Donors not only allowed me to develop these skills but also to experience such fulfilling emotions: the pride I had in Shaniyah when her math improved, the gratefulness I felt when she confided in me, the steadfast commitment I expressed to transplant patients, and the joy I had in collaborating with other passionate club members.

I envision a career as a physician to demand these skills of me and more, and I have confirmed my desire to become one after feeling so enriched by practicing them.

Here’s what makes this personal statement such a good example of what works:

  • Desirable qualities: The student clearly demonstrates qualities any school would want in an applicant: teachability, adaptability, leadership, organization, and empathy, to name a few. This again uses the “show, don’t tell” method, allowing the readers to understand the student without hand-holding.
  • Personalized storytelling: Many in the healthcare profession will connect with experiences like the ones expressed here, such as addressing patient concerns relationally or the lack of blood donors during the recent pandemic. The writer automatically makes a personal link between themselves and the admissions committees reading this statement.
  • Extensive (but not too long): Without feeling too wordy, this personal statement uses nearly all of the 5,300 characters allowed on the AMCAS application. There’s no fluff left in the final draft, only what matters.

Avoid These Common Mistakes

You can learn a lot from those personal statements. They avoid the most common mistakes that med school applicants make when writing the medical school personal statement.

Here are some things you should avoid in your personal statement if you want to be a doctor:

  • Name-dropping: Admissions counselors won’t be impressed when you brag about your highly regarded family members, associates, or mentors. You need to stand on your own feet — not someone else’s.
  • Dishonesty: Lies and exaggerations can torpedo your application. And they’re bad habits for anyone entering the medical field. Don’t do it.
  • Unedited AI content: Artificial intelligence can help you edit and improve your writing, but don’t let it do the work for you. Your statement needs to be authentic, which means in your voice! A chatbot can’t feel or adequately convey your own empathy, compassion, trauma, drive, or personality.
  • Grammatical errors and typos: Have someone reliable proofread your essay and scour it for typos, misspellings, and punctuation errors. Even free grammar-checking apps can catch mistakes!
  • Telling without showing: I’ll reiterate how important it is to prove your self-descriptive statements with real-life examples. Telling without showing won’t persuade readers.
  • Too many examples: Have 3-4 solid personal stories at most; only include a few that are crucial for providing your points. The more experiences you share, the less impact they’ll make.
  • Fluff and filler: Cut all fluff, filler words, and irrelevant points. There are many other places you can include information in your application, such as secondary essays on your clinical experience, volunteer work, and research projects . 

You can find more valuable do’s and don’ts in our in-depth guide to writing your best personal statement .

Need extra help? We’ve got you covered.

Schedule a meeting with medschoolcoach for expert support on writing and editing your personal statement. we’re here to help you impress medical school admissions committees .

Picture of Renee Marinelli, MD

Renee Marinelli, MD

Dr. Marinelli has practiced family medicine, served on the University of California Admissions Committee, and has helped hundreds of students get into medical school. She spearheads a team of physician advisors who guide MedSchoolCoach students.

See How We Can Help

Search for:, recent posts, medschoolcoach, recent blog posts.

Medical Schools Highest Average MCAT

What Medical Schools Have the Highest Average MCAT Score?

Getting into any US medical school is a huge accomplishment and extremely difficult to do. However, there are some schools[...]

November 28, 2017

High School Science Students

Spark Your Medical Dream Early: The High School Alliance of Future Physicians (HSAFP)

How would you like to start your medical journey before even stepping into college? Sounds exciting, right? This is exactly[...]

July 25, 2023

You can negotiate a medical school scholarship

Can You Negotiate a Medical School Scholarship?

When you receive a medical school acceptance, you are elated, and rightfully so! You've worked a tremendous amount to get[...]

March 7, 2018

The Pre-Med Journey

The Pre-Med Journey: What it Takes to Get into Medical School

Thinking about applying to medical school? Discover what high school students need to know about obtaining a career in medicine.

Successfully Planning for the USMLE Step 1 and 2 CK

Successfully Planning for the USMLE Step 1 and 2 CK

Get ready for the USMLE Step 1 and Step 2 with this free guide to study planning and resource utilization.

100 MCAT Study Tips

100 MCAT Study Tips

Taking the MCAT? These 100 tips and tricks will help you ace the MCAT.

Call us at (888) 381-9509

Call Us Now

Or, Schedule a Meeting Below

family medicine personal statement samples

Happy April Fool’s Day from MedSchoolCoach!

While mastering sleep-learning is still a dream, mcat go helps you study for the mcat while you are awake. listen to mcat go for free (a $99 value) by entering your email below to receive an exclusive discount code. this ain’t no joke..

family medicine personal statement samples

BrightLink Prep

Sample Personal Statement for Family Medicine Residency

family medicine personal statement samples

by Talha Omer, M.Eng., Cornell Grad

In personal statement | personal statement samples by field.

Personal Statement Prompt: A personal letter is required. We are looking for mature, enthusiastic physicians who bring with them a broad range of life experiences, are committed to providing excellent patient care, and can embrace the depth and breadth of experiences our program offers. In 500 words or less, please address the following areas:  What life experiences demonstrate your adaptability, team skills, leadership potential, and sense of compassion ?  Why do you feel the University of Manitoba would be your right choice?  In what ways are you connected to Manitoba?

Sample Personal Statement of a Physician

I was raised in a low-income household in Nigeria and experienced the struggles that came with financial hardship. One of the most impactful lessons I learned during this period was the importance of perseverance as my parents never gave up even when times were tough. Living with limited resources has helped me to develop a resilient spirit and be resourceful as I had to find creative ways to make things work with what was available. The skills acquired growing up helped me adapt easily while working in different hospital/clinic settings and with diverse patient populations.

Family medicine’s flexibility and diversity first drew me to the specialty, but my independent practice and varied patients’ interactions cemented my choice. I enjoyed interacting with and assessing patients’ social determinants during my independent practice and developing a healthcare plan. I remember a 44-year-old lady with chronic arthritis hip pain due to arthritis, but her financial situation and intolerable side effects prevented the prescription of an effective analgesic treatment. She was signed up for a program that covered her medications. I learned that social factors significantly affect people’s health and well-being.

I understood the significance of leadership when I led the anti-retroviral therapy (ART) team at Sango Primary Health Centre, I managed a team of nurses, community health workers, and volunteers with clear instructions in a supportive environment. I educated the community on HIV transmission and collaborated with NGOs to provide employment opportunities for HIV-positive patients who lost jobs due to discrimination. Patients’ HIV challenges/complaints were discussed amongst the team and questions from team members were asked/answered which helped us to achieve our individual and team goals. I also enjoy working in a team, I remember an instance when an auto accident victim was brought to my facility. After initial assessment and triaging, I worked with the nurses and community health workers to stop the bleeding and stabilize the patient, the pharmacist was consulted and the patient was given some analgesics before referring him to the teaching hospital because of the severity of his injuries as we didn’t have the resources to handle such a case at the health center. A nurse accompanied the ambulance and I followed up with calls to ensure the patients receive the needed care and treatment. I learned that effective collaboration with other healthcare professionals can help in giving patients optimal care.

My community-based experience as a licensed Clinical Assistant at Easton Medical Clinic Selkirk Manitoba has expanded my knowledge and clinical skills of the Canadian healthcare system, clinic management, and patient-centered care delivery. This experience has broadened my communication skills through interactions with patients and their families about their healthcare experiences, their concerns, and their fears were addressed in a non-judgmental manner

My career goal is to become a competent Family Physician, improving my patient health and well-being of individuals and underserved communities, and contributing to medicine advancement through lifelong learning, research, and quality improvement initiatives. The University of Manitoba’s Family Medicine Residency emphasizes community-based learning, making it my first choice as it will provide me with a deeper understanding of social determinants of health and the unique challenges patients face in underserved communities. This fits well with my prior experience as a GP that worked in primary health centers and aligns with my goal of improving the health and well-being of patients especially those in marginalized communities. I look forward to learning more about indigenous health, experiencing various learning opportunities in different clinic/hospital settings, and enhancing my skills in providing comprehensive, patient-centered care.

I have lived in Winnipeg since 2019 and I have enjoyed the vibrant city characterized by a stable economy and diverse population. I am committed to continuing to build our lives here. Outside work, I volunteer at the Inspire Community Outreach where I assist in providing support to those living with mental health challenges. This and other volunteer experiences taught me the importance of good relationships for mental well-being and community integration.

My diversity, resourcefulness, and experience as a General Practitioner from a resource-constrained country have prepared me well for residency training.

WANT MORE AMAZING ARTICLES ON GRAD SCHOOL PERSONAL STATEMENTS?

  • 100+ Outstanding Examples of Personal Statements
  • The Ultimate Guide to Writing a Winning Personal Statement
  • Common Pitfalls to Avoid in Your Personal Statement
  • Writing a Killer Opening Paragraph for Your Personal Statement
  • Ideal Length for a Graduate School Personal Statement
  • 100 Inspiring Quotes to Jumpstart Your Personal Statement

Sample Personal Statement for Masters in International Business

Sample Personal Statement for Masters in International Business My journey began amidst the kaleidoscope of Qatar's landscapes, setting the stage for a life attuned to cultural nuances. Transitioning to Riyadh in my teens, I absorbed a mosaic of traditions, sparking a...

Harvard Personal Statement Example

In this article, I will be providing a sample grad school personal statement for Harvard University. This example aims to show how prospective applicants like you can seamlessly weave your passion, skills, and relevant experiences into a compelling narrative. In...

[2024] 4 Law School Personal Statement Examples from Top Programs

In this article, I will discuss 4 law school personal statement samples. These statements have been written by successful applicants who gained admission to prestigious US Law schools like Yale, Harvard, and Stanford. The purpose of these examples is to demonstrate...

Sample Personal Statement Cybersecurity

In this article, I will be providing a sample grad school personal statement in the field of cybersecurity. This sample was written by an applicant who got admitted into George Mason, Northeastern and Arizona State University. This example aims to show how prospective...

How to Format & Structure Your Personal Statement for Grad School – The 5 Paragraph Approach

In the pursuit of graduate studies, one of the most critical aspects of the application process is writing a personal statement that truly reflects your passion, dedication, and individuality. While every applicant's journey is unique, structuring and formatting your...

WANT AMAZING ARTICLES ON GRAD SCHOOL PERSONAL STATEMENTS?

  • 100+ Personal Statement Templates

PersonalStatementMan

Join my mailing list, eras personal statement example - family medicine (june 2024).

Updated: Jun 12

Silly Spider-Man cartoon

The following ERAS personal statement example is for the medical residency application of Peter Parker, aka Spider-Man. I made it up completely and wrote it from scratch.

Below the example is a short discussion about the personal statement's elements and why I made many of the choices I made while writing it.

Need help with personal statement writing or any other aspect of your ERAS application? Feel free to check out all the services I offer . Reach out to me to request help today !

ERAS Personal Statement Example - Family Medicine

While volunteering at my aunt’s homeless shelter, I am frequently inspired by how much of an impact I can make with just a little bit of compassion and attention. Some of the relationships I’ve built there are years in the making. Others only span a single encounter. However, with them all, I learn to better collaborate with people toward making positive life changes. Each interaction is also an opportunity to emulate the family physician I have been inspired by all my life. You see, my aunt raised me. She has numerous chronic health concerns, so many of my afternoons growing up were spent accompanying her to the doctor’s office. Our family physician was always the rock we could rely upon. She still guides us through our ups and downs today. So, of course I strive to serve my patients with similar warmth, dedication, and expertise; my commitment to building trust and strong connections has defined my performance throughout rotations. Family medicine residency will be an extension of this, and I cannot wait to get started.

As a family medicine resident, I will relish cultivating lasting relationships with my patients as I treat them over months and years. I got a small taste of that kind of continuity of care during my recent family medicine elective after a 20-year-old female, diagnosed with pre-diabetes three months earlier, presented to our clinic. She was excited to share her progress, but dismayed when we found her weight and A1C levels had somehow increased. During my interview I quickly discovered she was confused. She had stopped “eating so much sugar” but was still consuming large amounts of pasta and soft drinks. Accordingly, with my attending’s permission, I took extra time to explain to our patient her condition in simple terms, print out an easy-to-follow DASH diet, and gain her commitment to healthier eating and regular exercise. Next, I reached out to a nutritionist and set her an appointment for later that week.

That day marked the first time I spoke with that particular nutritionist. Before my rotation was over, however, I scheduled many more appointments with him for others in similar situations. Meanwhile, I maintained weekly contact with my patient via phone and was present during her follow-up visits. The last time I saw the woman she had lost weight, brought down her A1C levels, and was glowing with pride for finally understanding and taking charge of her health. I only wish I could have remained her doctor long term. After all, my favorite part of my rotations were the relationships I built -- every time I saw an appreciative smile dawn on someone’s face, I’d remember my family physician and marvel at what she still means to my aunt and me. To play a similar positive role in a patient’s life, no matter how big or small, is a humbling and rewarding honor.

“With great power comes great responsibility,” my uncle once told me before he passed away. That’s my favorite quote. It means I must keep sharpening the knowledge and tools that I can use to benefit the patients I am so devoted to serving. Volunteer work is a large part of that journey, and my service is what initially inspired me to follow this path through medical school. I will surely continue to donate my time and energy whenever I can. I will do so locally, but I also intend to go abroad with an organization like Doctors Without Borders one day. Additionally, I envision starting my own family practice, possibly near my aunt’s homeless shelter. That’s far off in the future though, so, for now, I am seeking a family medicine residency program with great mentors and a focus on giving back to its surrounding community. As a grateful participant, I will always work hard and put my team and patients first.

Thanks for your time. It would be so awesome to swing by and meet you in person this interview season.

Spider-Man taking questions

At 661 words, Peter’s word count is ideal. Check out my article about optimal ERAS personal statement length  for further reading.

As far as structure, Peter follows my patented Cheeseburger Method, about which I go into great detail in my personal statement writing guide . It consists of three main elements:

Delicious cheeseburger

Element 1 (The Introduction or Top Bun ): His introduction discusses his background and motivation for pursuing medicine in the first place, as well as why he's applying for family medicine residency.

He mentions his family physician, but rather than just stating how great she is, he keeps the conversation about himself , discussing how he applies her lessons to his own day to day. This is critical -- in your personal statement, you must focus on YOU as much as possible.

Element 2 (The Middle or The Meat ): Next comes Peter’s patient story, which is the meat of the personal statement.

From his introduction, he transitions into the story by utilizing relationships and continuity. Then he describes a type of patient he’ll encounter all the time as a family medicine resident.

Through his patient story, without simply telling the reader, Peter shows   himself as:

1.       A good communicator and listener - He interviews his patient and quickly discovers she doesn’t grasp what constitutes a healthy diet.

2.       Proactive - Peter takes the initiative to ask his attending if he can spend extra time with the patient.

3.       Thorough - He goes way beyond just explaining the patient’s condition. He prints out clear instructions, confirms her understanding and commitment, and then reaches out to a nutritionist. Speaking of that…

4.       Proactive (again), thorough (yup, again), resourceful, and team-oriented - By establishing contact with the nutritionist, he creates a more all-encompassing, sustainable solution for his patient. He also begins a valuable relationship with a new team member.

5.       Dedicated and relationship-oriented - Peter follows up with his patient weekly and celebrates her progress when she comes back to the office.

All of the above attributes apply very well to family medicine. Great work Peter!!

Element 3 (The Conclusion or Bottom Bun ): Now Peter transitions into his conclusion by calling back to the introduction, reminding the reader of his family doctor and aunt. This brings everything full circle, which readers love.

Then, as he enters the final paragraph, he continues that theme by bringing in a quote from his uncle.

Note: I am usually not a fan of using quotes, but that’s just my preference and not a solid personal statement writing rule . And I couldn’t resist including this one because of its relevance to Spider-Man's origin story.

In his conclusion, Peter re-emphasizes his service work, touches on the future, and declares (being general so as not to alienate any programs) what he’s looking for in a residency program and what he will bring to the table as a team member.

Finally, to mitigate a bit of an abrupt ending, he adds a respectful line at the end about “swinging” into an interview. Get it?

Sorry, I really can’t help myself.

My full ERAS personal statement writing guide goes into great detail about my methods. And please feel free to check out all the services I offer . Reach out to me to request help today !

Photo credits:

Silly Spider-Man cartoon - https://depositphotos.com/portfolio-79260172.html

Spider-Man taking questions - https://depositphotos.com/portfolio-10128286.html

Cheeseburger - Abby Curtin

  • ERAS Personal Statement Examples
  • Residency Personal Statement
  • ERAS Residency Application

Recent Posts

How to Write a Residency Personal Statement (April 2024)

How to Find the Best ERAS Personal Statement Writing Service (April 2024)

Addressing Red Flags in Your ERAS Residency Application (April 2024)

mymedschool.org logo

Your cart is currently empty!

Family Medicine Personal Statements

Family Medicine Personal Statements

Sample Personal statement 1

Sample Personal statement 2

Sample Personal statement 3

Sample Personal statement 4

Sample Personal statement 5

Sample Personal statement 6

Sample Personal statement 7

Sample Personal statement 8

Sample Personal statement 9

Sample Personal statement 10

Sample personal statement 1 for Family medicine program

Although I began my medical career in anesthesia, I have developed a genuine interest in family practice during and after my anesthesia training. While practicing anesthesia, I always missed the wide spectrum of pathology, the continuous care for patients of different ages, and psychosocial issues in medicine. Family practice is the specialty that offers all these. Through family practice, I feel confident that I can achieve greater personal and professional satisfaction.

Although unable to find an observership program in family practice, I found several programs in general internal medicine. I have been doing observership at both Medical Education Alliance and Medical Hospital. During my observership training at both programs, I felt great achievement in using my knowledge of pharmacology, microbiology, and physiology to explain signs and symptoms of patients. This ensures me that I would be greatly benefited from a specialty in primary care.

I personally believe that providing long-term care, both preventive and therapeutic, to patients is an integral part of practicing medicine. During my observerships, I admired the relationship the attendings had formed with their patients. I love the idea of caring for patients from birth to death and building long-term friendships with them. While I was an anesthesiologist, my contact with most patients usually started one day before and ended one day after surgery. Even though patients developed complications from anesthesia, they did not seek medical care from anesthesiologists, because there was no communication channel between anesthesiologists and patients.

My knowledge and skills in anesthesia will be a useful asset in family practice. I am skilled in performing different kinds of invasive and non-invasive monitoring procedures. This will make me feel more comfortable while caring for critically ill patients. My hands-on clinical experience at both Memorial Medical Center of Medical University and the University of California was a unique opportunity to learn how the medical care system works and to enhance my communication skills with patients of all age groups. My volunteer experience at a mental health clinic helped me realize the importance of psychosocial issues in managing mental illnesses and develop a great compassion. By participating in all the activities during my observership training, I honed my history and physical skills. In addition, while I was an anesthesiologist, I enjoyed easing the anxieties of children undergoing surgery before I delivered anesthesia. This not only increased the quality of anesthesia care but also decreased the psychological harm on children. All these will make me better prepared for my residency training in family practice. Meanwhile I know exactly what I am going to face and I know I am able and willing to handle it.

I will bring enthusiasm, dedication, and professionalism, as well as clinical knowledge and skills to my family practice residency program. More importantly, I am very glad that I receive support from my family in applying for a family practice residency program. I seek a residency program that provides a balanced training approach. Through this training, I hope to develop excellent clinical skills and judgment and embark on a career with constant intellectual stimulation and the opportunity to provide long-term medical care to patients in a manner that I could not provide as an anesthesiologist.

Sample personal statement 2 for Family medicine program

“Did you hear about that huge earthquake in India?” asked one of my classmates who knew that I was from India. My jaw dropped and a hundred thoughts went through my head. There I was during my first year of medical school sitting anxiously about to take my first written anatomy exam when one of my classmates asked me this. I did not get a chance to listen to the news that morning, but there was a massive earthquake in the city where my family is from in India. This really hit a nerve. The anatomy test was the last thing on my mind at that point. Fortunately, later that night I found out all of my family was accounted for. That still didn’t put my mind at ease. The evening news showed the devastation the earthquake did to an already impoverished part of India. At that time, I felt more selfish than ever before. Here I was living this comfortable life in the U.S. and attending medical school with one of my biggest worries of the day being an anatomy test. I wanted to go to India and assist the injured, but I knew that my lack of medical training would not allow me to do that. Excluding this experience, I have always thought that it is a duty of mine to give back to the country where I am from. Being the first physician in my family and having this opportunity to give back to the place where my parents grew up has always been one of my major long term goals. Family practice will offer me the diverse knowledge base to treat many different types of ailments.

Attending an osteopathic medical school has offered me the opportunity to rotate through many months of family practice. When making my decision to enter this specialty, I asked myself what months during my clerkships I was happiest, which attending physicians I had the best interactions with, and what patient populations I liked the most. The answers to all these questions drew me to family practice. Although I formed meaningful relationships with the inpatient population during my internal medicine rotation and was able to acquire vital medical knowledge, I felt unfulfilled in regards to the doctor-patient interaction in that environment and really missed the continuity of care. In my family practice experience, there was an abundant amount of clinical knowledge to gain and I found myself attracted to the type of relationship my attendings had with their patients. I was truly envious of the continuity of relationships that my attendings had formed with so many of their patients. The “medical visit” in the family practice clinic amazed me because of the amount of psychosocial issues that would be discussed. This required the family practice physician to go beyond the medical aspect of the visit and truly treat the person as a whole. It is much easier to have a patient be compliant and more honest about their health when the patient feels more at ease with their physician.

The public aid clinic on Big City’s Westside was a place where I further realized the importance of the family physician. My volunteer experience in that clinic reinforced the fact that not every patient has access to a separate pediatrician, gynecologist, or geriatric specialist. My love for educating patients on topics such as hypertension, diabetes, cholesterol and preventative medicine was easily fulfilled in this type of setting. Another one of my passions is to also treat children Family medicine, offered me this opportunity. This is where I realized that I want part of my time to be involved with clinics that provide healthcare to the underserved in Big City. The great thing about practicing medicine in Big City is that there are so many well-established free medical clinics all over the city and suburbs that doctors can donate their time at. I am confident that the underserved areas of will have more than enough spots available for family practice physicians to donate their time.

The bottom line is that family practice is both exciting and versatile. When a family physician goes into work she does not know what to expect. She could have a day full of common colds, orthopedic issues, and newly diagnosed diabetics. One can take all these tasks and consider them overburdening or view them as challenges. I have always found it easy to adjust to a constantly changing environment During and after my training, I want to be the type of family physician that my loved ones would want not only as their doctor, but also as part of their family unit.

My determination, resilience, and self-confidence are foundations of my personality and character that will be part of me throughout my residency, and when I am practicing as a family physician. My goal throughout this application process has to find a family practice residency program that plays a vital role in the community and a program that shares a passion for teaching, mentoring and showing residents that family practice is not just a practice of medicine but also a practice of how to work with people. I look forward to starting my family practice residency at your institution with enthusiasm, a willingness to work hard, learn, teach, and most importantly, to identify with my future patients so I can offer them the healthcare they deserve from a family physician.

Sample personal statement 3 for Family medicine program
Sample personal statement 4 for Family medicine program

My life experiences have prepared me for a career in family medicine. I find such a field to require genuine interest, dedication, and the ability to communicate and relate to patients of all age groups. Upon entering medical school with an interest skin conditions, I assumed that dermatology was the field for me. I soon discovered, however, that I was interested in treating the whole person, that I wanted a career that treats the entire body, mind and spirit.

Working with patients of diverse ages and backgrounds is essential to my happiness. As a physician, I strive to positively impact my patient’s lives. Through volunteering for Hospice during college and medical school, and working in a nursing home in high school, I have enjoyed working with the elderly. In addition, I have spent 2 years as an elementary school reading tutor and volunteered for a year in the NICU and Children’s Medical Center at the University of Florida. Building a rapport with teenagers as a camp counselor has proven both rewarding and insightful. All encompassing, family medicine allows me the opportunity to work with the pediatric, adult, and geriatric populations. A vast and diverse field, family medicine provokes my interest and will continue to excite me throughout my career.

Numerous life experiences have prepared me for this path. Traveling across the country with 35 teenagers for Big Tours and working as a Resident Assistant in a college dormitory exemplify my ability to work well with others. The oldest of four children, I have taken on the role of responsibility and leadership in my life. On an individual level, I have mentored a young child for the past several years, which has impacted us both greatly. Extracurricular activities have helped to mold me in the person I am today. Outside of medicine, I have experienced whitewater rafting down the Snake River, water skiing on Lake Tahoe and hiking through Bryce Canyon. I am not only adventurous and easy to get along with but I am also a dedicated hard worker who thrives on patient care. Volunteering for several years in a family practice office has given me insight into the successful qualities of a family physician: patience, compassion and excellent clinical skills. My various work and volunteer experiences have confirmed that family medicine offers everything I want in a career – the ability to make a difference in the lives of patients while working in a field which I find exciting, challenging, and rewarding.

Much of my passion for family medicine stems from my desire to practice preventive medicine. The ability to retard or prevent the manifestations of disease inspires me. I intend to focus my future practice around this concept. Emphasis on treating the entire patient as a whole is also important to me. This holistic philosophy is essential to building strong relationships with patients. I want to become the best physician I can for my patients – a good listener, an excellent diagnostician, and a loyal confidant. To achieve these goals I have high expectations for my residency program. To develop the clinical skills essential to becoming an excellent family physician I must train at an institution dedicated to academic excellence and superior patient care. A strong academic setting combined with my desire to succeed will mold me into a well-trained physician. In the future, I desire to work in a clinical setting with an academic affiliation, which allows me the opportunity to teach what I have been taught to others. Many experiences in my life have helped me decide, unequivocally, that the field of family medicine is the path for me. My desire to treat and heal patients’ medical and social troubles drives me to become a successful physician. I look forward to the future with excitement and optimism as I enter the field of family medicine.

Sample personal statement 5 for Family medicine program

Since the age of 6 years old, I found myself frequently in the emergency room due to severe asthma. It was frightening as a child, but I clearly recall the warm and friendly doctors who comforted me. Since I grew up on Welfare and Medi-Cal, it meant sometimes having to wait long hours before getting medical attention. While waiting, I would chat with other patients and found that I was overwhelmed by the vast array of existing illnesses. Today I am no longer overwhelmed by the intricacy’s of medical diseases, but instead I’m inspired by the challenges of caring for a broad spectrum of various aliments. This is one of the reasons I have chosen family medicine.

I am a University of State graduate with a Bachelors of Science in Physiology. I participated in an internship at State Medical Center in the operating room where I found that I enjoyed easing the anxieties of patients’ pre and post surgery. In addition, I worked with a paraplegic for one year who I cared for daily. My patient was extremely frustrated because he had to be entirely dependent on me for many daily functions. However, there was no greater fulfillment than to see him grow comfortable and trusting toward me because I not only cared for him medically, but I took the time to establish a strong bond. This experience helped me develop a great compassion and understanding of the difficulties patients have co-existing with their physical disabilities. My desire to launch a positive role in a patient’s recovery along with the long term interactions established through continual care, have been other factors that have sparked my further interest in pursuing Family Medicine.

The goal of improving my Spanish contributed to my decision to attend medical school at the University in Guadalajara, Mexico. I had the fortunate experience of working with Spanish speaking patients in a family practice clinic. I was able to follow the entire family history of many patients throughout three years. Being able to become an integral part of these patients’ lives is another crucial factor that causes me to seek family practice. During my last year of medical school, I participated in the pre-internship program at the Mexican Institute of Social Services which provided me with outstanding training. I enjoyed rotating through all of the various rotations; especially pediatric, obstetrics, dermatology, and geriatrics. Family practice is well suited for me because it encompasses a multitude of areas in medicine. I want to continually be faced with the challenges that family medicine offers such as diagnosing a wide-range of diseases from different specialties, instead of focusing on one particular disease or organ system.

I also enjoy working with the elderly and learned some of their fears when I took care of my most important patient yet, my mother. I took a semester off to care for my mother who was diagnosed with metastatic colon cancer. Although it was extremely difficult emotionally watching her suffer daily, I knew that I had done my best in caring for her. I learned that many elderly patients neglect routine medical care and are hesitant or afraid to seek medical attention. As a consequence, they endanger their health or lives. My experiences have helped me mature greatly and have made me realize the critical responsibility as a primary care provider to continually teach preventive care.

I am applying to your residency program in family medicine because it is a well rounded community based program with broad-based training and a high level of responsibility that will prepare me to practice in settings comprised of a diverse population. In the future, I see myself as training future medical students. In addition, I plan to serve as a health advocate and mentor to children and adolescents. I feel that I will be an excellent role model for children, especially those in underserved communities, since I too grew up in an underserved area myself. I would like to return to my community or an underserved community where I feel that I am well prepared to fulfill the challenges and needs of my patients.

I am confident that I will benefit from your residency program because I have learned how to work and interact well with patients, families, doctors, and my colleagues through my life experiences. I also will never forget my earliest impressions of the dedicated and kind doctors from the emergency room. This is the type of Doctor I will be for all my patients

Sample personal statement 6 for Family medicine program

“ Some men see things as they are and say, WHY; I dream of things that never were and say, WHY NOT” –Robert Frost.

With this dream and vision along with many years of hard work and dedication, I am ready to embark in the exciting field of Family Medicine. Family Medicine adds a unique element to the practice of medicine that is very important: empathy. Family Physicians have compassionate beliefs and display caring values towards patient care. My choice to enter the area of Family Medicine expands this fascination and I am content, confident, and certain with this decision.

Beginning with grade school and throughout medical school, I made every effort to become the person and the physician I imagined. I learned early that two qualities produce a great physician: leadership and dedication. I realized ahead of time the importance of leadership and commitment as I participated in various sport activities in college. I enjoyed volunteering for my church in the community outreach program where I coordinated our youth group to assist various homeless shelters in Detroit. I later had the opportunity to be a teaching assistant for undergraduate and medical students in Anatomy class. I have gained many experiences by contributing to others the unselfish attitude, understanding, and dedication needed to be a respectful leader.

I was devoted to reach my aspirations of becoming a physician when I left my family and loved ones to attend the basic science portion of medical school abroad. This experience not only provided me with an outstanding and solid medical education, but also allowed me to mature and grow personally and professionally. I had the unique opportunity to discover how medicine was conducted in the Caribbean and to apply what I learned there to better my skills here at home. During my third year of clinical rotations in medical school, I was selected by the attending physician to lead my Internal Medicine group and to co-lead in my Surgery and Psychiatry clerkships. Furthermore, I was chosen by my Internal Medicine attending to work as a student assistant. My obligations were to admit, conduct a thorough physical exam and history, write progress notes, and dictate discharge summaries on detoxification patients. I learned to be proficient in patient management and to carry responsibilities in a structured and competent manner. These experiences have provided me the tools and knowledge to become the physician I want to be.

Many of the achievements I have attained are due to the encouragement and help of my family. Born in California and raised in New York, my parents have inspired in me the strong moral principles and loyalty that have made me the person I am today. Being the oldest of five children, our family was a close group largely because of our spiritual faith. Church service and fellowship was an integral part of our family upbringing and continues to be a vital part of my life. I am also exceptionally grateful for my wife for her immeasurable love and support.

Family Medicine best balances my abilities and assures my career ambitions. During my clinical rotations in Large City, I observed many aspects of medicine. I enjoyed working with patients in every age range and particularly enjoyed helping and solving their individual problems. The philosophy of Family Medicine is what especially appeals to me. It is important to me the idea of the continuity of care, being an active advocate, and most importantly to provide competent and complete health care. I desire to enter an exciting family practice residency program that will provide the needed training to work in a private practice or to teach in an academic or hospital setting. I also wish for a friendly and supportive environment where the faculty, residents, and staff work together as a collaborative team. Finally, I seek a residency program not only to facilitate my future career in medicine, but also would encourage me to continue to develop into the physician I envisioned. My greatest goal is to challenge myself to be the best physician I can be, without any regrets, and to “dream of things that never were and say, Why Not”.

Sample personal statement 7 for Family medicine program

As sage smoke rises from a shell full of embers, I watch Sandra waft the smoke over her body, washing away any reticence she had about sharing her past. She holds an eagle’s feather to help provide the strength to talk about  her struggles with alcohol and drugs . I witnessed her testimony in the Healing Circle, a Native-American tradition, adapted to treat clients at the Healing Center. Part of my Family Medicine rotation was spent studying the impact of cultural activities on drug and alcohol rehabilitation. While I had seen Maggie in clinic for her chronic joint pain due to rheumatoid arthritis and had discussed treatment options for her somatic complaints, I saw how the Healing Circle catered to her emotional and spiritual needs, giving her the courage to admit how badly alcohol and drugs had damaged her health, and to give her the resolve to abstain from further abuse.

Username or email  *

Password  *

Forgotten password?

[email protected]

+44 (0)20 8834 4579

Medicine Personal Statement Examples

Get some inspiration to start writing your Medicine Personal Statement with these successful examples from current Medical School students. We've got Medicine Personal Statements which were successful for universities including Imperial, UCL, King's, Bristol, Edinburgh and more.

Personal Statement Examples

  • Read successful Personal Statements for Medicine
  • Pay attention to the structure and the content
  • Get inspiration to plan your Personal Statement

Personal Statement Example 1

Check out this Medicine Personal Statement which was successful for Imperial, UCL, QMUL and King's.

Personal Statement Example 2

This Personal Statement comes from a student who received Medicine offers from Bristol and Plymouth - and also got an interview at Cambridge.

Personal Statement Example 3

Have a look at this Medicine Personal Statement which was successful for Imperial, Edinburgh, Dundee and Newcastle.

Personal Statement Example 4

Take a look at this Medicine Personal Statement which was successful for King's, Newcastle, Bristol and Sheffield.

Personal Statement Example 5

Pick up tips from this Medicine Personal Statement which was successful for Imperial, Birmingham and Manchester.

Personal Statement Example 6

This Personal Statement comes from a student who got into Graduate Entry Medicine at King's - and also had interviews for Undergraduate Medicine at King's, QMUL and Exeter.

Loading More Content

  • What is DPC?
  • Our Providers
  • Vaccines at SFM
  • Vaccination Guidance
  • Vaccination & Stem Cells
  • Patient Handbook
  • Dr. Story's Podcast
  • Ear Piercing
  • Forms and Agreements

	Story Family Medicine

Andrew Becker PA-C

Andrew Becker, PA-C, was born in Pullman, WA, and graduated from WSU with a degree in Sports Medicine in 2005. In 2010 he received his EMT certification and in 2011 he graduated from the University of Washington’s MEDEX program with a Master of Clinical Health Services degree. He has spent nearly 10 years back on the Palouse practicing internal medicine, urgent care, and wound care as a Physician Assistant. He also serves as a volunteer EMT in Moscow and as Athletic Trainer for Logos School. Andrew loves building lasting relationships with his patients and is looking forward to continuing to build relationships with patients on the Palouse at Story Family Medicine. 

family medicine personal statement samples

Dr. Matthew Rice

 Dr. Matthew Rice graduated from Oklahoma State University College of Osteopathic Medicine in Tulsa, OK in 1993. He also completed his Family Medicine Residency in Tulsa. DOs receive the full MD medical school education which additionally includes two years training in musculoskeletal treatment. DOs are also encouraged to view patients holistically (treating both body and soul/spirit). Dr. Rice is double board certified in Family Medicine and has a wide range of experience, having worked in private practice, various rural clinics, Indian Health Services, the Veteran’s Administration and as an ER physician. His practice philosophy is to treat his patients as he would his own beloved family. Speaking of which, he is married to his wonderful wife Doris, has two grown daughters and six delightful and unique grandchildren, whom he delights in!  

Dr. Matthew McCabe

   Dr. Matthew McCabe graduated with his Bachelor's degree from New Saint Andrews College, right here in Moscow. After a career as a teacher, he completed his MD degree at the University of Calgary and his residency training in Family Medicine at the University of Alberta. He has practiced family medicine in a variety of settings including with the Canadian Armed Forces and has provided hospital and ER coverage in rural Alberta. Dr. McCabe is looking forward to growing his practice through care and skill. 

Dr. McCabe cares for patients of all ages.

Lisa Halpert, NP

Meet our Nurse Practitioner, Lisa Halpert. Lisa has worked primarily in internal medicine and urgent care. 

Lisa is upbeat, loves a good cup of coffee, and is ready to lend a patient and attentive listening ear. While she primarily works alongside Dr. Story to support his panel, she is available on a case-by-case basis for well-woman exams of all our providers' patients.

Dr. Rod Story

Dr. Rod Story is a highly trained, Board-Certified family physician.  He has provided medical care for 18 years in many settings, including the ER, hospital, hospice, and nursing homes. His broad experience and skills can provide for most of your medical needs, including office procedures and advanced testing. 

In 2017 Dr. Story received the Physician Excellence Award from Pullman Regional Hospital.

Story Family Medicine.  All rights reserved.

Powered by GoDaddy Website Builder


See more in , , , , ,

-->
| | |
FAMILY PRACTICE RESIDENCY PERSONAL STATEMENT

 


The Medfools Personal Statement Library is now open!

These sample personal statements are here for your viewing pleasure (fully anonymous). We're hoping to add more in the future, including Pre-Med personal statements. If you've got one to add to the free library, don't forget to .

FAMILY MEDICINE RESIDENCY PERSONAL STATEMENT

My interest in family medicine has developed from personal and professional experiences over a long period of time. My father being a family physician was my first source of inspiration. I spent hours watching him at work and was impressed with bond he had with his patients and their families.
My interest in family practice was further heightened from my experiences in medical school. Volunteer work in underserved areas and clinical rotations during the last two years of medical school exposed me to a wide variety of cases from the prenatal through the geriatric stages of life. I realized that managing the health care of patients in these different age groups is challenging and intellectually appealing to me. I also felt that familiarity with the family system is essential in performing complete assessment and treatment of a patient's condition. Family practice provides exposure to the full spectrum of problems and issues in medicine and provides an opportunity not only to connect with a patient but also with their families. Family practice gives scope of practice to the needs of the community, puts me on the front line of diagnosis and planning, and helps nurture long-term relationships with patients. For this reason, family practice is the most comprehensive and most attractive type of medicine to me.

Most importantly, I realized it would suit me perfectly. Having taken care of my grandmothers during their end stages of life and being a mother of two, I have developed tremendous mental and physical stamina, learned to think quickly and remain focused, and work with patience and diligence. My fine eye for detail, interpersonal skills, and enthusiasm to work with a wide variety of patients will come a long way to mold me into an empathetic and successful family physician.

Throughout my well-rounded and diverse education I have tried to keep an open mind and expand my horizons. The decision to come to the US for graduate medical education was a step in that direction. My internship training after medical school had provided me hands-on clinical experience to wide variety of cases in all facets of medicine. The exposure I gained as an observer in pediatrics and internal medicine, as a research assistant and as a hospice care volunteer has further honed my clinical skills and added to my personal growth and social awareness. During this time, I attended daily rounds with residents, participated in taking medical history, discussed differential diagnosis and management, interpreted lab values and regularly attended noon conferences and grand rounds. My research has led to three articles, one of which has been accepted for publication by the Journal of the American College of Cardiology. I continue to work as an observer at UMDNJ pediatrics emergency department while I prepare for Step 3.

I seek a residency program that will provide intensive clinical experience with a diverse patient population, promote patient education and prevention, encourage participation in community service, and requires significant resident responsibility for patient care. Though I welcome the excitement and intellectual challenge, what I want the most from my career is to channel a lifetime of energy into something meaningful and worthwhile. I am thoroughly committed to pursue a career in family medicine and I can only hope that I am able to give the profession as much as I think it can give me.

I am applying to a family practice residency program with a strong belief that it would bring the best of me and help me achieve my future goals. I will bring energy, enthusiasm, integrity and creativity to the program and will be committed to my work, fellow staff and to all my patients. After completion of residency, I plan to pursue fellowship in geriatrics. My ultimate goal is to dedicate myself to serve an underserved population while committing time to research and education in geriatrics.

IMAGES

  1. Family Medicine Residency Personal Statement: Samples & Help

    family medicine personal statement samples

  2. With this site you can get your personal statement for family medicine

    family medicine personal statement samples

  3. Family medicine personal statements helps and tips on this site will

    family medicine personal statement samples

  4. 77+ Personal Statement Examples

    family medicine personal statement samples

  5. Family Medicine Residency Personal Statement Writing

    family medicine personal statement samples

  6. 10 Best Personal Statement Template

    family medicine personal statement samples

COMMENTS

  1. Family Medicine Residency Personal Statement Examples

    It is a good idea to review family medicine residency personal statement examples as you prepare for residency applications. Whether you are applying through ERAS (in the United States) or CaRMS (in Canada), your residency personal statement is one of the most important application components that you will need to submit. Residency program directors and admission committees want to get a sense ...

  2. Impressing: Personal Statement

    The best personal statements are memorable. They paint a picture in the mind of the reader and tell a story about who you are, how you got here, and where you want to go. The personal statement is vitally important because it is frequently used to help determine who gets interviewed and ranked. Overarching theme: Look over your CV and think ...

  3. PDF Family Medicine Personal Statement

    Family Medicine Personal Statement . The first pivotal moment in my medical career occurred when I was 18 and met my friend and college roommate who was diagnosed with Type I Diabetes at the age of two. During the summer after my first year in medical school, I volunteered as medical staff at the American ...

  4. PDF Personal Statement Family Medicine

    Personal Statement - Family Medicine. John L. Clark, Jr., MD practiced pediatric medicine in my hometown of Odessa TX. He was the person my parents came to trust and seek for answers when my health or my sibling's health was compromised. My parents built a long lasting and trusting relationship with this person; that relationship was ...

  5. Writing a Personal Statement

    Personal Statement. The personal statement is an important component of your application. While it's impossible to know the exact "weight" that a specific examiner will give to this aspect of your application, best estimates range from 5-25%. This is less than the relative contribution of your grades or Boards scores but a sizable chunk ...

  6. Family Medicine Residency Personal Statement: Samples & Help

    Our medical residency personal statement samples are done according to the admission board requirements, including format, style, word count, etc. Papers are written to represent individuals as perfect candidates for the chosen family medicine program. When another sample is ordered, our experts work on it from scratch, writing it according to ...

  7. Sample Family Medicine Residency Personal Statement

    These sample Family Medicine personal statement examples are here for your viewing pleasure (fully anonymous). We're hoping to add more in the future, including Pre-Med personal statements. If you've got one to add to the free library, don't forget to contribute yours. Although I began my medical career in anesthesia, I have developed a ...

  8. Family Medicine Residency Personal Statements

    The Medfools Family Medicine Sample Residency Personal Statement Library is now open! These sample Family Medicine personal statement examples are here for your viewing pleasure (fully anonymous). We're hoping to add more in the future, including Pre-Med personal statements. If you've got one to add to the free library, don't forget to contribute yours. FAMILY MEDICINE RESIDENCY PERSONAL

  9. Family Med Residency Personal Statements

    The Medfools Family Medicine Sample Residency Personal Statement Library is now open! These sample Family Medicine personal statement examples are here for your viewing pleasure (fully anonymous). We're hoping to add more in the future, including Pre-Med personal statements. If you've got one to add to the free library, don't forget to contribute yours. My path to family […]

  10. Writing a Personal Statement for Residency Application

    For the moment, forget everything you know about writing histories and physicals. While preparing your personal statement: Avoid abbreviations. Avoid repetitive sentence structure. Avoid using ...

  11. Residency Personal Statement Examples

    FAMILY MEDICINE RESIDENCY PERSONAL STATEMENT EXAMPLE. "Yup, this is what I want to do!". Is what 10-year old Yenchi thought after her well child visit. I became interested in family medicine simply because I wanted to be like my family medicine physician. She became my career inspiration.

  12. 20+ Residency Personal Statement Examples

    Commentary on Residency Personal Statement Example #2. "Medicine is not a job, it is a way of life.". As the son of a cardiothoracic surgeon, my father's mantra constantly echoed in my mind. I was raised in an environment where sacrifice and duty were familiar concepts from a young age.

  13. 3 Medical School Personal Statement Examples [2024 Update]

    Example 3 — Beyond the Diagnosis: The Importance of Individualized Care in Medicine. The applicant who wrote this personal statement was accepted into Touro College of Osteopathic Medicine and Nova Southeastern University College Of Osteopathic Medicine. Dr. Haywood sighs and shakes her head upon opening the chart.

  14. Sample Personal Statement for Family Medicine Residency

    Sample Personal Statement of a Physician. I was raised in a low-income household in Nigeria and experienced the struggles that came with financial hardship. One of the most impactful lessons I learned during this period was the importance of perseverance as my parents never gave up even when times were tough. ... Family medicine's flexibility ...

  15. ERAS Personal Statement Example

    ERAS Personal Statement Example - Family Medicine (June 2024) Updated: Jun 12. The following ERAS personal statement example is for the medical residency application of Peter Parker, aka Spider-Man. I made it up completely and wrote it from scratch. Below the example is a short discussion about the personal statement's elements and why I made ...

  16. Family Medicine Personal Statements

    As a family physician, I feel I have the best opportunity to impact a person's overall well-being and at the same time further enrich my life with culture and diversity. Sample personal statement 8 for Family medicine program. My interest in family medicine has developed from personal and professional experiences over a long period of time.

  17. Family Medicine Residency Personal Statement Sample

    The Medfools FP Medicine Sample Residency Personal Statement Library is now open! These sample Family Medicine personal statement examples are here for your viewing pleasure (fully anonymous). We're hoping to add more in the future, including Pre-Med personal statements. If you've got one to add to the free library, don't forget to contribute yours. "Did you hear about […]

  18. 6 Real Examples Of Successful Medicine Personal Statements

    Personal Statement Example 6. This Personal Statement comes from a student who got into Graduate Entry Medicine at King's - and also had interviews for Undergraduate Medicine at King's, QMUL and Exeter. Get some inspiration for your Medicine Personal Statement with these successful examples from current Medical School students.

  19. PDF Writing the Personal Statement

    The Personal Statement Should: •Use one or two specific incidents to show what has been learned from these experiences. •Discuss the experiences, people and events that influenced your decision to become a physician or prepared your to enter this field. •Discuss your motivation for medicine

  20. User ID (Email)

    Billing: Effective December 1, 2018, you will receive statements from Gritman Medical Center: Moscow Family Medicine Clinic for all services rendered after 12/1/2018. If you have any questions, please call our business office at 208-882-2011. We support our patients having access to their health information through our Patient Portal.

  21. Our Providers

    Dr. Rod Story. Dr. Rod Story is a highly trained, Board-Certified family physician. He has provided medical care for 18 years in many settings, including the ER, hospital, hospice, and nursing homes. His broad experience and skills can provide for most of your medical needs, including office procedures and advanced testing.

  22. Family Medicine Personal Statement, Example Family Medicine ...

    These sample personal statements are here for your viewing pleasure (fully anonymous). We're hoping to add more in the future, including Pre-Med personal statements. ... My interest in family medicine has developed from personal and professional experiences over a long period of time. My father being a family physician was my first source of ...

  23. Sports Medicine Clinic

    Call to schedule an appointment 208-882-0540. "We want to keep people active, healthy and exercising as much as possible and as long as possible," Dr. Christensen said. "You are going to get good, high-quality care at our Sports Medicine Clinic.". Dr. Jacob K. Christensen. Teen and adolescent wellness exams with sports physicals.

  24. My Doctor

    Effective December 1, 2018, you will receive statements from Gritman Medical Center: Moscow Family Medicine Clinic for all services rendered after 12/1/2018. If you have any questions, please call our business office at 208-882-2011. API: We support our patients having access to their health information through our Patient Portal.